Respiratory Pops Questions

Ace your homework & exams now with Quizwiz!

D. 71 y.o. past smoker who quit 10 years ago with 35 pack years with COPD.

2. Identify the patient that is a candidate for lung cancer screening: A. A 49 y. o. smoker with 45 pack years with no medical problems. B. 73 y.o. past smoker who quit 20 years ago with 50 pack years with diabetes mellitus and hypertension. C. 62 y.o. smoker with 30 pack years with metastatic colon cancer (no other life limiting illnesses). D. 71 y.o. past smoker who quit 10 years ago with 35 pack years with COPD. E. 55 y.o. smoker with 19 pack year with coronary artery disease.

B. Surgery then Chemotherapy Correct answer is B. Treatment of Non-small Cell Lung Cancer: Stage 1: Surgery Stage 2: Surgery then Chemotherapy Stage 3: Chemotherapy plus Radiation then maybe surgery Stage 4: Chemotherapy

4. A 75 y.o. male presents with Stage 2 Lung Adenocarcinoma. What are the treatment recommendations? A. Surgery B. Surgery then Chemotherapy C. Chemotherapy then Radiation D. Chemotherapy

D. Chemotherapy

4. A 75 y.o. male presents with Stage 4 Lung Adenocarcinoma. What are the treatment recommendations? A. Surgery B. Surgery then Chemotherapy C. Chemotherapy then Radiation D. Chemotherapy

c. Yes, a venous blood gas is adequate to evaluate pH

1. A 25-yr old Type 1 diabetic patient presents to the Emergency Department (ED) with acute onset of abdominal pain, lethargy and purulent nasal drainage. Her symptoms have developed over the last two days. Oxygen saturation by pulse oximetry is 96% on room air. The nurse has placed an intravenous catheter and is going to draw blood to send to the lab. She asks you if you want her to send a venous blood gas in addition to electrolyte and blood glucose levels. The BEST response to the nurse is: a. No, only an arterial blood gas can measure bicarbonate b. No, only an arterial blood gas is adequate to evaluate pH c. Yes, a venous blood gas is adequate to evaluate pH d. Yes, a venous blood gas is adequate to evaluate oxygenation

b. Asthma Answer B is correct. This patient has obstruction on spirometry as evidence by a reduced FEV1/FVC ratio below 70%. The obstructive lung diseases includes Asthma, COPD and Bronchiectasis. Obstructive lung disease if defined by their difficulty exhaling due to bronchoconstriction (asthma/COPD), excess mucus (asthma/COPD/Chronic bronchitis/bronchiectasis), airway collapse due to loss of tethering (COPD/Emphysema) and/or loss of elastic recoil (COPD/Emphysema). This patient has significant response to bronchodilators with a > 10% increase in FVC or FEV1. The combination of obstruction and improvement with bronchodilators makes Asthma the most likely diagnosis. COPD and Bronchiectasis don't frequently respond to bronchodilators. As he is a non-smoker, COPD is even more unlikely. Bronchiectasis frequently presents predominantly with a productive cough. Congestive heart failure would present with restrictive physiology on spirometry.

1. A 35-yr-old presents with dyspnea on exertion that occurs when he plays basketball and engages in heavy physical exertion at work. He has a small amount of sputum production and occasional cough. He is a lifelong non-smoker. Spirometry is done and his FEV1 is reduced at 75% predicted with a FEV1/FVC ratio of 67% (LLN 72%). His FEV1 is improved by 15% following inhalation of albuterol. What is the most likely diagnosis? a. Bronchiectasis b. Asthma c. COPD d. Congestive heart failure

a. Abnormal post-translational processing of a transmembrane protein

1. A 4-year-old boy is brought to the office for recurrent "pneumonias". He is small for his age. Sweat chloride testing reveals a high amount of chloride. Which of the following abnormalities is most likely seen in the patient? a. Abnormal post-translational processing of a transmembrane protein b. Decreased proteasome activity c. Defective channel formation on the cell membrane d. Decreased transcription of a transmembrane protein.

b. Decreased osmotic pressure Answer B is correct. This patient likely has decreased osmotic pressure from poor protein intake from malnutrition and lack of production in cirrhosis. Decreased lymphatic drainage occurs when the lymph system is impaired in some way. Common etiologies include lymphoma leading to disease in the thoracic lymph nodes as well as traumatic damage to the thoracic duct. Increased hydrostatic pressure is typically due to left heart failure resulting in increased pressure build up in the lungs. Increased vascular permeability occurs with lung injury such as infection, connective tissue disease and malignancy.

1. A 43-year-old female with a history of hepatitis C and cirrhosis presents with a large right-sided pleural effusion. On exam, she appears malnourished and emaciated. What is the most likely mechanism of this pleural effusion? a. Decreased lymphatic drainage b. Decreased osmotic pressure c. Increased hydrostatic pressure d. Increased vascular permeability

D. Reassure patient that no additional follow-up is needed

1. A 60 y.o. male with 5 pack years of smoking presents with a 4 mm right upper lobe solitary pulmonary nodule. What is the next most appropriate step in evaluation? A. Repeat CT scan in 12 months B. PET scan C. IR guided biopsy D. Reassure patient that no additional follow-up is needed

B. Assess pre-test probability of malignancy Answer B is correct. A Solitary pulmonary nodule calculator can be used to assess the risk of malignancy. In this case, the risk of malignancy is approximately 85%. If the patient is a surgical candidate then lobectomy is recommended. A repeat CT scan in 12 months is too long of follow-up for high-risk patient. Initiating chemotherapy and radiation needs tissue diagnosis and staging prior to starting therapy. Fleischner Society 2017 Guidelines < 6 mm ignore the nodule 6-8 mm repeat imaging at defined intervals > 8 mm assess pre-test probability Solitary Pulmonary Nodule (SPN) Risk Assessment (ACCP Guidelines, Gould - Chest 2013): Pretest probability of malignancy from SPN calculator: < 5% = serial CT imaging with interval determined by size of nodule (Fleischner Guidelines) 5-65% = PET-CT or biopsy PET scanning provides 2 answers: Is nodule hypermetabolic/concerning for cancer? Where has the disease has spread to? PET-CT: Sensitivity for SPN = 95%. False positive for SPN = 15-25%. PET scan pitfalls: < 1 cm nodules have poor sensitivity; false positive in infection, granuloma and inflammation; false negative in carcinoid, slow growing adenocarcinomas and hyperglycemia. IS NOT HELPFUL IN GROUND GLASS NODULES because they are not solid enough to consistently become PET avid. >65% = Resection surgically (preferred, greatest chance for cure)

1. A 70 y.o. male with 100 pack years of smoking presents with a 25 mm right upper lobe spiculated solitary pulmonary nodule. What is the next most appropriate step in evaluation? A. Repeat CT scan in 12 months B. Assess pre-test probability of malignancy C. Initiate chemotherapy and radiation

d. Intensive care unit Answer D is correct. Using the CURB-65 tool, this patient has at lead 3 markers for severity. Confusion, Respiratory rate > 30 and Age ≥ 65 are the criteria this patient has met. Blood pressure and Blood urea nitrogen level are not reported. If a patient has ≥ 3 points then they should be cared for in the intensive care unit due to their excess mortality.

1. A 75-year-old male is brought into the emergency room with confusion and shortness of breath. His wife states he has been having fevers and productive cough over the last couple of days. Vital signs are significant for fever to 39.5 C, tachycardia to 110 beats/minute and tachypnea to 35 breaths/minute. On exam he is mildly confused but pleasant and has crackles over his left lung base. Chest x-ray shows a left lower lobe pneumonia. In addition to empiric antibiotics where should this patient receive care? a. Home b. Skilled nursing facility c. Inpatient hospital ward d. Intensive care unit

d. pH Answer D is correct. Venous pH is approximately 0.04 units more acidotic then arterial pH. A normal arterial pH is 7.4. A normal venous pH is 7.36. Other measured values in venous blood gases have a wider variance and are less predictable. PvCO2 (venous PCO2) reflect the metabolic state of the vein from which the blood was obtained. Because blood is usually collected from the arm, the VBG mostly reflects the metabolic state of the arm, which is of limited clinical utility. Oxygenation varies widely in the venous system and is related to the oxygen extracted by the tissue.

1. Which of the following venous blood gas results correlates BEST with the result obtained on a simultaneous arterial blood gas? a. PCO2 b. PO2 c. Oxygen saturation d. pH

B. 71 y.o. past smoker who quit 5 years ago with 55 pack years with hypothyroidism and hyperlipidemia.

2. Identify the patient that is a candidate for lung cancer screening: A. 58 y.o. smoker with 50 pack years with ischemic cardiomyopathy and LVEF 15%. B. 71 y.o. past smoker who quit 5 years ago with 55 pack years with hypothyroidism and hyperlipidemia. C. 55 y.o. smoker with 19 pack year with no medical problems. D. 74 y.o. past smoker who quit 30 years ago with 80 pack years with diabetes mellitus and hypertension. E. 49 y.o. smoker with 55 pack years with COPD.

b. IL-5

2. A 25 y.o. female presents with intermittent shortness of breath yearly during the spring and fall. She does not normally have dyspnea during her daily activities but currently she is moderately short-of-breath. Physical exam reveals expiratory wheezing. Spirometry reveals an FEV1/FVC ratio of 65%. FEV1 is < LLN at 72% predicted and improves to 89% predicted after albuterol. Based on her likely diagnosis which substance is playing a role in her disease? a. IL-1 b. IL-5 c. Interferon gamma d. TNF

b. Staphylococcus aureus Answer B is correct. The initial presentation is consistent with influenza viral infection. Typical symptoms of influenza include abrupt onset fever, headache, myalgia, and malaise. Symptoms usually improve over 5-7 days. This case describes a "double sickening" which is a patient was getting better but then developed new or worsening symptoms. Influenza infection increases the risk of secondary Staphylococcus infection. This is due to the viral-induced to the mucociliary clearance. Staphylococcus aureus is a Gram-positive cocci that occurs in clusters. It is a rare cause of community acquired pneumonia but is common after influenza and as a hospital acquired pathogen. Cytomegalovirus (CMV) is typically asymptomatic in immunocompetent individuals but in immunocompromised host CMV can cause pneumonitis along with a variety of other syndromes including retinitis, hepatitis, colitis and meningoencephalitis. CMV also plays a big role in chronic lung transplant rejection. Histologically CMV displays intracellular inclusions of viral particles called Owl eyes. Mycobacterium tuberculosis is an acid-fast bacilli and is not associated with post-viral illness presentation. Klebsiella pneumoniae is a typical community acquired pneumonia which could fit the clinical picture; however, Klebsiella is an encapsulated, lactose fermenting, Gram-negative rod (not a Gram-positive cocci in clusters).

2. A 25-year-old medical student develops fever, myalgias and a non-productive cough. The myalgias are so severe she says "I feel like I got hit by a truck." She misses class for a few days but the myalgias and fevers resolved. Five days later, the cough becomes productive and now she is dyspneic with regular activity. A chest radiograph demonstrates bilateral infiltrates. Gram stain is shown. Which of the following is the most likely pathogen to be isolated from the patient's sputum? a. Cytomegalovirus b. Staphylococcus aureus c. Mycobacterium tuberculosis d. Klebsiella pneumoniae

c. Panacinar Answer C is correct. This patient has obstructive lung disease on PFTs as evidence by a reduced FEV1/FVC ratio. He has evidence of hyperinflation with an elevated TLC. He has a reduced DLCO. DLCO is reduced in COPD due to destruction of the alveoli and the alveolar capillaries. The diagnosis of COPD can be made based on the non-reversible obstruction on PFTs and the presence of emphysema on chest imaging. Chronic Obstructive Lung Disease (COPD) is a clinic syndrome that encompasses both emphysema and chronic bronchitis. Emphysema is the destruction of the alveoli seen on radiology and pathology. Chronic bronchitis is excess mucus production and inflammation in the airways with symptoms of chronic productive cough. Alpha-1-antitrypsin deficiency is a genetic cause of emphysema and presents with early onset emphysema. It is inherited in an autosomal recessive fashion. Pathologically Alpha-1-antitrypsin deficiency patients have panacinar emphysema. Emphysema in smokers typically occurs in the upper lobe and is centriacinar. Centriacinar refers to the center of the secondary pulmonary lobule. Centriacinar pattern of damage is seen due to smoking affecting the airway located in the center of the lobule. Panacinar emphysema is due to damage of all of the secondary pulmonary lobule simultaneously ("Pan-" means "all"). Additionally centriacinar emphysema typically has a predilection for the upper lobes, which is where the cigarette smoke goes. Contrast that with panacinar emphysema, which is commonly seen throughout the entire lung, including the bases, as seen in this case. Alpha-1-antitrypisin protein is the major serum inhibitor of neutrophil elastase. Neutrophil elastase is found in neutrophils and macrophages and is responsible for the destruction of elastin. When neutrophil elastase is left uninhibited, neutrophils and macrophages destroy the alveolus resulting in COPD. In this case, smoking history is not provided but smoking in the setting of Alpha-1-antitrypin deficiency dramatically increases the risk of COPD. Paraseptal emphysema is also commonly caused by smoking but as its name implies, is along the outer rim of the lung. Pneumothorax is a common complication of paraseptal emphysema due to its location near the pleural lining. Subcutaneous emphysema can occur in a variety of clinical scenarios including skin infection with gas formation, pneumomediastinum where air tracks along the subcutaneous tissue planes and iatrogenically from a misplaced chest tube.

2. A 45-yr-old white male complains of dyspnea and cough that are increasingly interfering with his ability to do physical activity required in his job. He is adopted and does not know his genetic background. He has basilar (base/lower portion of the lung) emphysema on chest radiograph and pulmonary function test shows FEV1 45% predicted, FEV1/FVC ratio of 55%, TLC 125% predicted and DLCO 30% predicted. There was no improvement in spirometry after bronchodilators. Given this patient's likely diagnosis, what is the most likely type of emphysema? a. Paraseptal b. Centriacinar c. Panacinar d. Subcutaneous

c. Protein 4.2 g/dL Answer C is correct. The Pleural protein/Serum protein ratio of 4.2/4.5 is greater than 0.5 (Answer C) and thus defines this as an exudate. Light's Criteria (only need to meet one): Protein Ratio >0.5 LDH Ratio > 0.6 LDH >2/3 upper limit of normal The pleural LDH/serum LDH is 130/270 is less than 0.6 which does not meet criteria for an exudate. The pH and WBC do not differentiate exudative from transudative effusions.

2. A 65-year-old male had coronary artery bypass grafting 3 weeks ago and presents with increasing dyspnea. Chest radiograph shows a moderate left-sided pleural effusion. Laboratory results are as follows: WBC 11/mm3 (Normal 6-12 mm3), Serum LDH 270 U/L (Normal 98-192 U/L), Serum Protein 4.5 g/dL (Normal 5.9-7.5 g/dL), and thoracentesis results in 500 mL of bloody fluid. Which of the following pleural fluid results would characterize this as an exudate? a. pH 7.37 b. LDH 100 U/L c. Protein 4.2 g/dL d. WBC 1200/mm3

b. Acute on chronic respiratory acidosis

4. A patient presents with somnolence and an ABG is obtained showing: pH = 7.29, PaO2 = 60, PaCO2 = 64, HCO3- = 32. (Normal values; pH 7.35-7.45, PaO2 80-100, PaCO2 33-45, HCO3- 22-28)? What is the acid-base status? a. Acute respiratory acidosis b. Acute on chronic respiratory acidosis c. Chronic respiratory acidosis d. Metabolic alkalosis

d. This P/F ratio indicates severely impaired oxygenation

2. In the treatment of Acute Respiratory Distress Syndrome (ARDS), the PaO2/ FiO2 (P/F) ratio is used to determine treatment options. What does a P/F ratio of 95 tell you about a patient? a. This is a normal ratio and demonstrates normal oxygenation b. The A-a gradient should be used instead of the P/F ratio as the A-a gradient is more linear than the P/F ratio c. The P/F ratio is not as accurate as the A:a gradient to assess oxygenation while on supplemental oxygen d. This P/F ratio indicates severely impaired oxygenation

b. He has a metabolic acidosis Answer B is correct. The primary acid base disorder is identified by the direction the pH moves. In this example, the pH is acidotic (< 7.4) and the bicarbonate is low indicating a metabolic acidosis. There is a concurrent respiratory compensation which can be mistaken for a respiratory alkalosis; however, in a true respiratory alkalosis the compensation is in excess of what one would expect. To calculate the appropriate compensation for metabolic acidosis, Winter's formula is used. Expected PCO2 = (1.5 x HCO3) + 8 ± 2. Expected PCO2 in this case = (1.5 x 4) + 8 = 14 ± 2 so this patient is compensating appropriately.

2. Your patient has the following arterial blood gas values: pH = 6.92, PaO2 = 90, PaCO2 = 16, HCO3- = 4 (Normal values; pH 7.35-7.45, PaO2 80-100, PaCO2 33-45, HCO3- 22-28) Which of the following statements regarding the primary acid-base disorder is correct? a. He has a respiratory acidosis b. He has a metabolic acidosis c. He has a respiratory alkalosis d. He has a metabolic alkalosis

a. Haemophilus influenza Answer A is correct. Haemophilus influenza is a "typical" bacterial pathogen in pneumonia and is also commonly seen in otitis media. Streptococcus pneumonia could have a very similar presentation with pneumonia and otitis media but was not given as an option. Haemophilus influenza (H.flu) is an oxidase-positive, Gram-negative rod that requires chocolate agar with factor V (NAD) and X (hematin) to grow. H.flu Type B is encapsulated by a polysaccharide capsule prevents that phagocytosis. Because of this polysaccharide capsule a Haemophilus influenzae Type b (Hib) vaccine is available. However, Hib accounts for only 10% of H.flu related acute otitis media in children with the rest resulting from non-typable (non-encepasulate) H.flu. The utility of Hib vaccination comes from its ability to prevent Hib associated meningitis. Staphylococcus aureus is a Gram-positive cocci that occurs in clusters that grows readily in standard culture medial. It is commonly associated with hospital acquired pneumonia. Pseudomonas aeruginosa is an aerobic non-lactose fermenting, oxidase positive, Gram-negative rod that is a common hospital acquired pathogen. Neither Staphylococcus nor Pseudomonas are commonly associated with otitis media. Influenza virus aka "The Flu" is an orthomyxovirus. It causes a clinic syndrome of abrupt onset fever, headache, myalgia, and malaise. Symptoms usually improve over 5-7 days. Outbreaks of influenza frequently occur. When influenza virus causes a pneumonia it is typically with an "atypical" pattern of diffuse infiltrates rather than a lobar pattern as seen here.

3. A 10-year-old presents to her pediatricians office with fever, ear pain, cough and shortness of breath. She is up to date on all her vaccinations. There is no rash. On exam she has a bulging left tympanic membrane and crackles heard over the right middle lobe. Chest x-ray shows a right sided infiltrate. Sputum culture is obtained but does not grow any pathogens. a. Haemophilus influenza b. Staphylococcus aureus c. Pseudomonas aeruginosa d. Influenza virus

a. Negative methacholine challenge

3. A 20-year-old medical student presents with intermittent shortness of breath and is sure she has asthma. She reports the symptoms do not have a link to specific events or times of the day. Physical exam is normal. Spirometry reveals an FEV1/FVC ratio of 82%. Which of the following is most helpful to exclude asthma as a diagnosis? a. Negative methacholine challenge b. Normal chest x-ray c. Absence of allergen identification on skin prick testing d. Normal peripheral blood count without eosinophilia

a. Intermittent Answer A is correct. The severity of asthma is determined by symptoms, nighttime awakenings, use of a short-acting beta agonist (SABA), baseline lung function (FEV1) and exacerbation risk. This patient has symptoms less than twice a week (intermittent), has no nighttime symptoms (intermittent), has no interference with normal activity (intermittent) and her FEV1 is 84% predicted (intermittent). Exercise induced symptoms do NOT contribute to the severity scoring of Asthma. Asthma is graded base on the worst component of severity. All four of this patient's components of severity are in the intermittent range so her level of severity is intermittent. She has no history of exacerbations requiring oral corticosteroids, which also places her in the intermittent category.

3. A 26-yr-old medical student with known asthma develops dyspnea when running long-distances or when exposed to cats. She has symptoms less than twice a week, never wakes with dyspnea and uses her albuterol once or twice a month on average. FEV1 is 3.43 L (84% predicted). What is the severity of her asthma? a. Intermittent b. Mild persistent c. Moderate persistent d. Severe persistent

b. LDH of 275 U/L Answer B is correct. Almost all parapneumonic effusions are extudative so an Pleural LDH/Serum LDH > 0.6 and Pleural Fluid LDH > 2/3 the upper limit of normal in lab does not differentiate simple verses complicated parapneumonic effusions. A complicated parapneumonic effusion is characterized by the presence of any of the following: septations on ultrasound, pH < 7.2, glucose < 60, positive gram stain or culture. Empyema is characterized by frank pus coming out of the chest.

3. A 56-year-old male presents with a pneumonia and pleural effusion related to Streptococcus pneumoniae infection. Ultrasound and diagnostic thoracentesis are performed. Laboratory results are as follows: Serum LDH 250 U/L (98-192), Serum Protein 6.5 g/dl (5.9-7.5). Which of the following pleural fluid finding would help establish the diagnosis of simple parapneumonic effusion? a. Multiple septation seen on ultrasound b. LDH of 275 U/L c. pH 7.15 d. Gram stain positive for gram positive cocci e. Pus seen on thoracentesis

c. Acute respiratory alkalosis

3. A patient presents with respiratory failure and is intubated and placed on mechanical ventilation in the ED. The ventilator is placed in volume control mode with settings of tidal volume of 500 mL, respiratory rate of 28, positive end-expiratory pressure of 5 cm H2O and FiO2 of 0.5. ABG reveals pH = 7.52, PaO2 = 350, PaCO2 = 25, HCO3- = 24. (Normal values; pH 7.35-7.45, PaO2 80-100, PaCO2 33-45, HCO3- 22-28)? What is the acid-base status? a. Acute on chronic respiratory acidosis b. Acute respiratory acidosis c. Acute respiratory alkalosis d. Chronic respiratory alkalosis

C. Small cell carcinoma

3. Which cancer is most closely associated with smoking? A. Adenocarcinoma B. Carcinoid C. Small cell carcinoma

B. Squamous cell carcinoma Correct answer is B. Squamous cell carcinoma is strongly associated with smoking. Small cell carcinoma is also strongly associated with smoking. Adenocarcinoma is increased in smokers but is not as tightly associated as Squamous cell and Small cell carcinomas. Carcinoid is not associated with smoking.

3. Which cancer is most closely associated with smoking? A. Adenocarcinoma B. Squamous cell carcinoma C. Carcinoid

d. She has alveolar hyperventilation Answer D is correct. The pH is alkalotic and the PaCO2 is low indicating the process is respiratory. The bicarbonate level is normal suggesting an acute process. The only clinical scenario, of the answers suggested, that results in a respiratory alkalosis is alveolar hyperventilation. Examples of alveolar hyperventilation include anything that can cause tachypnea (fast breathing) like anxiety (claustrophobia), acute pain, high altitude (the mild hypoxia increases respiratory drive), and pulmonary embolism. Vomiting induces a metabolic alkalosis due to loss of stomach acid. Acute renal failure causes a metabolic acidosis. Heroin is an opiate that suppresses respiratory drive causing a respiratory acidosis.

3. Your patient has the following arterial blood gas values: pH = 7.50, PaO2 = 75, PaCO2 = 27, HCO3- = 24 (Normal values; pH 7.35-7.45, PaO2 80-100, PaCO2 33-45, HCO3- 22-28)? Which of the following statements is correct? a. She has been vomiting for the past 3 days b. She has acute renal failure c. She overdosed on heroin d. She has alveolar hyperventilation

d. Streptococcus pneumoniae Answer D is correct. Patient has a right upper lobe infiltrate on chest x-ray and sputum showing Gram-positive diplococci in the setting of symptoms of pneumonia with fevers, cough and dyspnea. All of these would indicated a "typical" pneumonia caused by Streptococcus pneumoniae. Streptococcus pneumoniae is the most common bacterial pathogen isolated in community acquired pneumonia. It is encapsulated by a polysaccharide capsule to prevent phagocytosis. There are > 90 different polysaccharide capsules resulting in different strains of the organism. The commonly administered pneumococcal vaccines have 13 and 23 of the most common strains. Patients with anatomic (surgically removed) or function (sickle cell) asplenia (without a spleen) are at increased risk of encapsulated organisms like Streptococcus pneumoniae, Haemophilus influenzae Type b, Neisseria meningitides and Capnocytopaga becoming disseminated. Emergent treatment is indicated in this patient. Legionella pneumophila is an intracellular, weakly Gram-negative rod that requires charcoal yeast extract agar buffered with increased iron and L-cysteine to grow in culture. Legionella can be seen on silver stain which is usually used to identify fungus. Legionella outbreaks are common due to its aerosol transmission usually from a contaminated water source. Hyponatremia and diarrhea are commonly associated with Legionella infections. Chlamydia pneumoniae and Mycoplasma pneumoniae presents as an "atypical" community acquired pneumonia and are not seen on Gram stain.

4. A 30-year-old female presents with fever to 39o C, cough productive of rusty brown sputum, dyspnea and shaking chills. Sputum is sent for Gram stain and chest radiograph is shown. Patient has a right upper lobe infiltrate on chest x-ray. Her only past medical history is surgical removal of her spleen after a car accident. What is the most likely organism causing this pneumonia? a. Chlamydia pneumoniae b. Legionella pneumophila c. Mycoplasma pneumoniae d. Streptococcus pneumoniae

c. Eosinophilic cationic protein Answer C is correct. This clinical scenario describes the late phase of an asthma attack or exacerbation. In the early phase of an asthma exacerbation, the mast cell releases histamine, which is the main mediator of bronchoconstriction. Four to six hours later the late phase of an asthma exacerbation is mediated by release of inflammatory mediators from eosinophils. Specifically eosinophilic cationic protein is released causing bronchial irritability. Macrophage metalloprotease and neutrophil elastase are released in response to bacterial infections and hydrogen peroxide, an oxygen radical, is produced by the neutrophils in the setting of infection.

4. A 40-yr-old male with history of allergic rhinitis and asthma presents with worsening dyspnea. He had been raking leaves earlier in the day and had to use his albuterol inhaler at the time. He initially got better but 4 hours later, he developed more wheezing and worsening dyspnea. What are the most likely mediators of his current symptoms? a. Neutrophil elastase b. Hydrogen peroxide c. Eosinophilic cationic protein d. Macrophage metalloprotease

d. Severe persistent

4. A 43-yr old female with known asthma presents for evaluation of dyspnea. She states she wakes 2 times per month with dyspnea. She has symptoms daily. She uses her albuterol inhaler 3 times per day. Clinical assessment of inhaler technique reveals good performance. She states her asthma limits her physical activity some of the time. Her FEV1 is greater than 80% of predicted value. What is the clinical severity of her asthma? a. Intermittent b. Mild persistent c. Moderate persistent d. Severe persistent

b. Chylothorax Answer B is correct. This case describes a new onset lymphoma with fevers, night sweats and weight loss. He also has enlarged lymph nodes in his chest. His pleural effusion is milky weight and triglycerides are > 110 mg/dL making it a chylothorax. Chylothorax are frequently cause by disruption of the lymphatic system inside the chest. Trauma to the thoracic duct and lymphoma are the two most common causes. An empyema is frank pus coming out of the pleural space which can be white. The lack of neutrophils on cell count and the triglyceride abnormality makes empyema unlikely. Empyema falls on a spectrum of disease. Simple parapneumonic effusions are exudates that are causes by capillary leak from infected lung. Complicated parapneumonic effusion are the next step up on the spectrum and are exudative plus any of the following pH < 7.2, glucose < 60 mg/dL, positive gram stain or positive culture. Empyema represents the worst disease on the spectrum. Malignant pleural effusions are exudative and can be cloudy in nature but rarely appear purulent. Pleural fluid cytology has an initial sensitivity of approximately 60% and a second sampling will add another 15%. The remaining malignant pleural effusions require more invasive testing to diagnose.

4. A 50-year-old male presents with shortness of breath. He has associated fevers, night sweats and weight loss. Chest x-ray reveals a right sided pleural effusion with an associated bilateral hilar and mediastinal lymphadenopathy. Ultrasound guided thoracentesis is performed and reveals thick white drainage. Gram stain is negative for bacteria. Cell count reveals a lymphocyte predominance. Triglycerides are 205 mg/dL. Cytology is negative. What is the most likely diagnosis? a. Empyema b. Chylothorax c. Malignant pleural effusion d. Simple parapneumonic effusion

c. pH = 7.33, PaCO2= 50, HCO3- = 26 Neuromuscular weakness results in alveolar hypoventilation. The inability to take a deep breath (lower tidal volume) results in reduction in minute ventilation (tidal volume x respiratory rate). Lower minute ventilation raises the PaCO2 which lowers the pH. Every 10 mm Hg increase in PaCO2 the pH decreases by 0.08. For example, if a PaCO2 increases from 40 to 50 then the pH will decrease from 7.40 to 7.32. Examples of neuromuscular weakness include Myasthenia gravis, Guillan-Barre syndrome, muscular dystrophy or amyotrophic lateral sclerosis (Lou Gehrig's disease)

4. Which set of arterial blood gas values is consistent with Myasthenia Gravis, a neuromuscular disease causing weakness of the respiratory muscles (Normal values; pH 7.35-7.45, PaCO2 33-45, HCO3 22-28)? a. pH = 7.50, PaCO2= 32, HCO3- = 24 b. pH = 7.39, PaCO2= 24, HCO3- = 14 c. pH = 7.33, PaCO2= 50, HCO3- = 26 d. pH = 7.45, PaCO2= 45, HCO3- = 30

d. Secondary spontaneous pneumothorax Answer D is correct. The patient has underlying lung disease of cystic fibrosis. This places him at risk of secondary spontaneous pneumothorax. Other causes of secondary pneumothorax are relate to an underlying disease such as emphysema, tumors in the lung, chest trauma, esophageal rupture, catamenial, PCP pneumonia and cystic lung disease. Primary spontaneous pneumothorax occurs in patients without underlying pulmonary disease. To diagnose a tension pneumothorax there must be a buildup of pressure due to the pneumothorax. This would result in pushing of the mediastinal contents away from the pneumothorax. In this case there is no shift in the mediastinum so tension pneumothorax is unlikely. The pressure that builds up prevents venous return to the heart and is associated with low blood pressures and developing shock. Pneumomediastinum develops when air tracks into a mediastinal plane. This can originate from a pneumothorax, a ruptured esophagus or from the upper airway/neck. Commonly, subcutaneous air is also seen in pneumomediastinum with crepitus felt in the neck and over the upper chest wall.

5. A 12-year-old boy with cystic fibrosis presents with new on set right sided chest pain and shortness of breath. Chest x-ray reveals chronic bronchiectasis and right sided pneumothorax without mediastinal shift. What is the most likely diagnosis of this clinical finding? a. Tension pneumothorax b. Pneumomediastinum c. Primary spontaneous pneumothorax d. Secondary spontaneous pneumothorax

d. Add inhaled fluticasone Answer D is correct. This patient has mild persistent asthma. Please note that exercise induced symptoms and albuterol use with exercise are not used as part of the grading of asthma. The severity of asthma is based on the worst component in severity. In this case once a week nocturnal awakenings (mild) is the most severe symptom. If more than once a week, than that would be moderate. In general, nocturnal awakenings of asthma are a red flag symptom that patient's asthma is not well controlled. His daytime symptoms are borderline intermittent/mild at 2-3 times/week with the remainder "intermittent" (No interference in normal activities, Normal lung function testing). Based on the diagnosis of mild persistent asthma the recommendations would be to up-titrate his therapy from Step 1 (intermittent short acting beta-agonist) to Step 2 (initiate low dose inhaled corticosteroids like fluticasone). Once a patient has persistent asthma, a controller medication (one that decreases inflammation) is a key component of their therapy. Oral theophylline is listed as an alternative in Step 2, 3 and 4; however, theophylline has a narrow therapeutic window, several potential toxicities and is not recommended over inhaled corticosteroids. Theophylline is a phosphodiesterase inhibitor which increases cyclic AMP resulting in bronchodilation. Increasing albuterol frequency would not represent a step up in therapy, which is indicated in this case. Omalizumab is an anti-IgE monoclonal antibody, which is considered in refractory asthma as part of Step 5 and 6.

5. A 22 year old male with a history of asthma presents with increasing dyspnea. He was diagnosed with asthma 2 years ago and was given albuterol as needed for his symptoms. He exercises 5 days per week and gets dyspnea and cough each time. He uses his albuterol before and after exercise to treat his symptoms. Over the last several months, he has noted increased dyspnea even when not working out. He notices shortness of breath with wheezing two to three times per week aside from exercise. He is also having shortness of breath that wakes him up once a week. He has not limitations to his daily activities. Spirometry reveals an FEV1 of 83% predicted. What is the next most appropriate step in the management of his asthma? a. Add subcutaneous Omalizumab b. Increase albuterol frequency c. Add oral theophylline d. Add inhaled fluticasone

d. Montelukast

5. A 30-year-old male with history of asthma and chronic rhinosinusitis presents with clinic with increased shortness of breath. He recently sprained his ankle and has been taking aspirin for the pain. He has been taking his fluticasone/salmeterol inhaler regularly. On exam, he has rhinorrhea, bilateral nasal polyps and bilateral wheezing. In addition to stopping aspirin, what other therapy would most likely benefit this patient? a. Discontinue salmeterol b. Anti-IgE therapy c. Macrolide antibiotic (Azithromycin) d. Montelukast

B. Small round blue cells with nuclear molding

5. A 74 y.o. female presents with weight gain and increased facial hair. She has noticed increasing abdominal girth and fuller face when she looks in the mirror. She denies headache. Vital signs are significant for BP of 160/95. Patient was noted to have a mass on CXR and she underwent a biopsy. What is the most likely histopathologic pattern seen? A. Intracellular bridges and keratinization B. Small round blue cells with nuclear molding C. Gland formation with invasion

C. Gland formation with invasion

5. A 75 y.o. female presents with erythematous rash on her face and around her neck. She also has weakness in her arms and difficulty rising from a chair. CPK is elevated on lab. Patient was noted to have a mass on CXR and she underwent a biopsy. What is the most likely histopathologic pattern seen? A. Intracellular bridges and keratinization B. Small round blue cells with nuclear molding C. Gland formation with invasion

a. pH = 7.50, PaO2 = 60, PaCO2 = 32, HCO3- = 24 Answer A is correct. High altitudes have a lower inspired pressure of oxygen than at sea level. The fraction of inspired oxygen (FiO2) is the same 0.21 (21%) at all altitudes, but the atmospheric pressure reduces at higher altitudes. The body's initial response to abrupt ascent to high altitude is alveolar hyperventilation. Answer B demonstrates a metabolic acidosis (low HCO3-) and respiratory compensation (blowing off CO2). Answer C demonstrates an acute on chronic respiratory acidosis. Any time there is a change from the normal pH (7.35-7.45) there is an acute process going on. The acute process is a respiratory acidosis with a high PaCO2. There also appears to be a metabolic alkalosis with high HCO3-, but this actually represents metabolic (renal) compensation for a chronic respiratory acidosis. Another way to conceptualize this process is if PaCO2 acutely increases from 40 to 70 (increase in 30 = 10 x 3) then you would expect the pH to drop from 7.40 to 7.16 (decrease in 0.08 x 3 = 0.24) but since the pH is higher, the kidneys have had some time to compensate. Answer D demonstrates a metabolic alkalosis as seen in vomiting and renal losses of acid.

5. A healthy 40 y.o. male sets out to climbs Mount Everest and after 24 hours he gets an ABG drawn. Which set of arterial blood gas values best represent those obtained? (Normal values; pH 7.35-7.45, PaO2 80-100, PaCO2 33-45, HCO3- 22-28)? a. pH = 7.50, PaO2 = 60, PaCO2 = 32, HCO3- = 24 b. pH = 7.36, PaO2 = 80, PaCO2 = 24, HCO3- = 14 c. pH = 7.26, PaO2 = 60, PaCO2 = 70, HCO3- = 31 d. pH = 7.47, PaO2 = 90, PaCO2 = 42, HCO3- = 35

a. Rhinovirus Answer A is correct. Rhinovirus (aka the common cold) is a picornavirus. A "cold" is more precisely called an upper respiratory tract infection (URI). Classic symptoms include fever, rhinorrhea (nasal drainage), cough, and sore throat. Symptoms typically last for 5-7 days. There are numerous viruses that can cause upper respiratory tract infections including: Rhinovirus, Coronavirus, Adenovirus, Influenza, Parainfluenza and Respiratory syncytial virus. These same viruses are also commonly isolated in patients at the same time they are diagnosed with community acquired pneumonia. They also contribute to exacerbation in asthma and COPD. Haemophilus influenzae is a cause of bacterial pneumonia and otitis media and does not typically cause URI symptoms. Cytomegalovirus (CMV) is typically asymptomatic in immunocompetent individuals but in immunocompromised hosts CMV can cause pneumonitis along with a variety of other syndromes including retinitis, hepatitis, colitis and meningoencephalitis. Coxsackievirus is a member of the Enterovirus family and is the causative agent in hand, foot, and mouth disease. Hand, foot, and mouth disease is a highly contagious syndrome which includes fever, oral vesicles/blisters on the buccal (cheeks) mucosa and tongue, and peripherally distributed small, tender cutaneous lesions on the hands, feet, buttocks, and genitalia.

5. An 18-year-old patient with a history of asthma presents with increasing shortness of breath. She also has a runny nose and a sore throat. She has been around friends that also have "colds". On exam, she has wheezing bilaterally, inflamed nasal mucosa and erythematous tonsillar pillars. What is the most likely cause of her illness? a. Rhinovirus b. Haemophilus influenza c. Cytomegalovirus d. Coxsackievirus

c. Opiate overdose

5. An ABG is obtained from a 36-y.o. male, on room air, at sea level, in the Emergency Department. Which of the following is the most likely etiology of this ABG: pH = 7.24, PaO2 = 65, PaCO2 = 58, HCO3- = 24, O2 Saturation 88%? (Normal values; pH 7.35-7.45, PaO2 80-100, PaCO2 33-45, HCO3- 22-28)? a. Pulmonary embolism b. Diabetic ketoacidosis c. Opiate overdose d. Anxiety

b. Neutrophils

6. A 78-year-old female present with increased shortness of breath. She is a lifelong smoker with over 100 pack years. PFTs reveal a reduced FEV1/FVC ratio of 58%. Chest CT scan shows centriacinar, upper lobe predominant emphysema and hyperinflation. What are the cells most responsible for her disease? a. Eosinophils b. Neutrophils c. Lymphocytes d. Club cells

a. Chronic respiratory acidosis

6. A patient with very severe COPD presents with the following ABG: pH = 7.38, PaO2 = 78, PaCO2 = 53, HCO3- = 32. (Normal values; pH 7.35-7.45, PaO2 80-100, PaCO2 33-45, HCO3- 22-28)? What is the acid-base status? a. Chronic respiratory acidosis b. Acute on chronic respiratory acidosis c. Acute respiratory acidosis d. Metabolic alkalosis

a. Catamenial Answer A is correct. The patient has chronic abdominal pain related to periods which may be endometriosis. Endometriosis occurs when endometrial lining tissue implants outside the uterus. With each cycle the tissue grows and then dies with hormonal changes. The endometrial tissue can move into the pleura via diaphragm fenestrations (holes) and implant onto the lung. When the tissue involutes during a period and pneumothorax can develop. This patient has no history of lung diseases making cystic fibrosis and asthma unlikely. Apical blebs are the typical pathogenesis of a primary spontaneous pneumothorax. The most peripheral and apical alveoli are under the most tension from the chronic elastic pull of the lung and are at risk of stretching into blebs that rupture. Young, tall, males are at the highest risk of developing primary spontaneous pneumothorax from apical blebs.

6. A 25-year-old female with history of chronic abdominal pain related to her periods presents to the emergency room with sharp chest pain and shortness of breath. She has no history of breathing difficulties. She is found to have a left sided pneumothorax on chest x-ray. What is the most likely etiology of her pneumothorax? a. Catamenial b. Cystic Fibrosis c. Apical blebs d. Status asthmaticus

a. Blastomyces dermatitidis Answer A is correct. Blastomyces is an endemic fungal organism found in organic materials East of the Mississippi. It is also carried by many animals like dogs and horses. Symptoms of infection include flu-like illness (fevers, chills, myalgias and headache) or pneumonia. In immunocompromised hosts, Blastomyces can cause disseminated disease with fevers, night sweats, weight loss, cough, skin rash with pustules or ulcers and lytic bone lesions. Histologically, it is described as a single "broad-based budding" yeast. When the Blastomyces yeast is budding/dividing, the connection between the cells is wide and there is only one at a time. Blastomyces is also dimorphic. This means it is a round yeast when at body temperature and mold hyphae at lower temperature. Other dimorphic fungus include Sporothrix, Coccidioides, Histoplasma, and Paracoccidioides. Cryptococcus neoformans causes pneumonia and meningitis in immunocompromised patients. Cryptococcus is characterized by narrow based budding and a very thick capsule that is highlighted by India ink staining. Histoplasma is smaller than Blastomyces and is found intracellularly within macrophages. Histoplasma is frequently asymptomatic in immunocompetent hosts but can become disseminated in the setting of immune suppression. Histoplasma capsulatum is found in the Missouri and Ohio River basins and is found in bird and bat droppings. Mycobacterium tuberculosis is an acid-fast organism that can cause pneumonia and lung cavities.

6. A 32-year-old male presents with recurrent cough and sputum production. He has had multiple courses of antibiotics following a chest radiograph that demonstrated a left lower lobe pneumonia. His symptoms would improve after antibiotics but would return within a week. History is significant for a summer vacation in Wisconsin where he was working around a family cabin. His current radiograph is shown. Sputum silver stain reveals yeast with single broad-based buds. What is the most likely diagnosis? a. Blastomyces dermatitidis b. Coccidioides immitis c. Histoplasma capsulatum d. Mycobacteria tuberculosis

D. SVC syndrome

6. A 60 y.o. male presents with increasing shortness of breath, facial swelling and headache. He admits to 75 pack years of smoking. Physical exam shows fullness and redness of the face and dilated chest wall veins. What is the most likely cause? A. Pericardial effusion B. Pleural effusion C. Main-stem bronchus occlusion D. SVC syndrome E. Superior sulcus tumor

C. Main-stem bronchus occlusion

6. A 60 y.o. male presents with increasing shortness of breath. He admits to 75 pack years of smoking. Patient is tachypnic with respiratory rate of 24. Physical exam shows dullness to percussion on the right, decreased breath sounds on the right and tracheal deviation to the right. What is the most likely cause? A. Pericardial effusion B. Pleural effusion C. Main-stem bronchus occlusion D. SVC syndrome E. Superior sulcus tumor

d. Tiotropium Answer D is correct. This patient has COPD as evidence by non-reversible obstruction on PFTs and reduced DLCO. The reduced DLCO is due to neutrophilic destruction of alveoli and their capillaries seen in emphysema. COPD treatment is targeted at bronchodilation. Treatment targets bronchodilation in COPD to relieve symptoms, which is very different from the approach used to treat asthma. Asthma treatment is targeted at treatment and prevention of airway inflammation. Tioptropium is a long acting muscarinic antagonist (LAMA) bronchodilator. Salmeterol and formoterol are examples of long acting beta-2 agonists (LABA) bronchodilators. Both LAMAs and LABAs are first line therapy for COPD. In Asthma, inhaled steroids are first line therapy due to their anti-inflammatory effects. Prednisone is an oral corticosteroid and is used to treat acute exacerbations of both COPD and Asthma but is not indicated for chronic management. Theophylline is an oral phosphodiesterase inhibitor, which increases cyclic AMP resulting in bronchodilation. Theophylline has a narrow therapeutic window, several potential toxicities and is not recommended over LAMA, LABA or ICS in the management of COPD.

6. A 70-year-old presents to clinic with shortness of breath and chronic productive cough. He coughs up sputum multiple times a day almost every day for the last 2 years. He has been smoking two packs of cigarettes per day for 40 years (80 pack years). His chest x-ray reveals hyperinflation. Spirometry reveals an FEV1/FVC ratio of 60% and FEV1 68% predicted. Bronchodilators did not change his spirometry. DLCO is reduced at 50% predicted. Which medication would be the best initial step in treatment? a. Fluticasone b. Theophylline c. Prednisone d. Tiotropium

b. pH = 7.15, PaCO2 = 95, HCO3- = 32 Answer B is correct. Any time there is a change from the normal pH (7.35-7.45) there is an acute process going on. Answer B demonstrates some degree of metabolic compensation with a rise in serum bicarbonate, which explains the chronic component. If we were to correct Answer B back to a normal pH using a change in pH of 0.08 = a change in PaCO2 of 10, then you would see that pH = 7.15 + (0.08 x 3) = 7.39 with PaCO2 95 - (10 x 3) = 65. A PaCO2 of 65 clearly demonstrates a chronic respiratory acidosis. Chronic respiratory failure is fully compensated with a normal pH, elevated PaCO2, and elevated bicarbonate. Answer A represents an acute metabolic acidosis without appropriate respiratory compensation. Respiratory compensation should occur very quickly in a normal patient. This should raise the concern for a hypoventilatory state. Answer C represents an acute respiratory acidosis without change in metabolic status. The metabolic compensation typically takes several days to develop. Answer D represents a mixed disorder. A mixed disorder occurs when the pH is normal (7.40) but there are two other identifiable acid-base disorders present. In this case, there is a metabolic alkalosis and a respiratory acidosis.

6. Which set of arterial blood gas values is consistent with Acute on Chronic Respiratory Failure (Normal values; pH 7.35-7.45, PaCO2 33-45, HCO3- 22-28)? a. pH = 7.17, PaCO2 = 40, HCO3- = 14 b. pH = 7.15, PaCO2 = 95, HCO3- = 32 c. pH = 7.32, PaCO2 = 48, HCO3- = 24 d. pH = 7.40, PaCO2 = 50, HCO3- = 30

a. Sweat chloride test Answer A is correct. Cystic fibrosis often presents with clinical features of sinopulmonary disease (bronchiectasis and chronic sinusitis) and gastrointestinal abnormalities (pancreatic insufficiency, meconium ileus, and failure to thrive). Bronchiectasis is abnormal, permanent dilation of airways (CT scan shown below). This patient's clinic presentation suggests CF. Cystic fibrosis is an autosomal recessive disorder that results in defects of chloride transport (Cystic Fibrosis Transmembrane conductance Regulator = CFTR) in the respiratory, gastrointestinal and reproductive tract. Sodium chloride is normally secreted into the airways, biliary tree, intestines, pancreatic ducts and sweat ducts to provide osmotic pressure to bring hydration into the area (Simply, salt pulls water into the area). When chloride transport is impaired, water does not enter and secretions become thicker. The thick secretions cause obstruction of the organ lumen resulting in end-organ damage (bronchiectasis, liver disease, constipation, pancreatic insufficiency in the form of malabsorption of fats and fat-soluble vitamins as well as diabetes mellitus). Diagnosis is confirmed with sweat chloride test > 60 mmol/L (below is a figure helping explain chloride shifts in CF). In patients with normal or borderline sweat chloride test CFTR mutation analysis is performed. In the United States, newborn screening is performed for cystic fibrosis.

7. A 12 y.o. male comes to the clinic for 6 months of weight loss and fatigue. His parents note loose stools and frequent bronchitis. On exam, he has crackles heard over multiple areas of both lungs. CT scan of the chest and abdomen show bronchiectasis and pancreatic fatty infiltration and atrophy. Which of the following is most likely to establish the diagnosis in this patient? a. Sweat chloride test b. Fecal fat testing c. Serum immunoglobulin quantification d. Sputum smears with acid-fast staining

A. Thymoma

7. A 40 y.o. female presenting with progressive weakness. She has difficulty rising off a chair and combing her hair. At the end of the day, she also gets double vision. She has high levels of circulating anti-acetylcholine receptor antibodies. A CT scan shows an anterior mediastinal mass. What is the most likely diagnosis? A. Thymoma B. Teratoma C. Thyroid Cancer D. Terrible Lymphoma

a. Actinomyces Answer A is correct. Actinomyces is a slow growing Gram-positive anaerobic bacteria that colonizes the mouth, colon and vagina. Actinomyces infections frequently lead to cervicofacial abscesses and fistulas. When aspiration occurs the Actinomyces can cause a pneumonia. Alcoholism is a risk factor for aspiration and subsequently actinomycosis. Histologically, Actinomyces is a unique organism in that it is a filamentous, branching Gram-positive rod. The material that comes out of the abscesses is frequently described as sulfur granules for their yellow color and gritty texture. The treatment of actinomycosis is with an extended course of penicillin. Mycobacterium tuberculosis is an acid-fast organism that can cause pneumonias and lung cavities. Mycobacterium tuberculosis can cause scrofula which is an infection of the cervical lymph nodes and could mimic the physical exam listed here. Streptococcus pneumoniae is the most common bacterial pathogen isolated in community acquired pneumonia and typically appears as Gram-positive diplococci. It also commonly causes otitis media which can cause swollen cervical lymph nodes but isn't associated with fistula development. Pseudomonas aeruginosa is an aerobic non-lactose fermenting, oxidase positive, Gram-negative rod that frequently causes hospital acquired pneumonias.

7. A 50-year-old male presents with increasing shortness of breath and cough. He states he has been dealing with an oral abscess for a couple of weeks now and recently it has started to ooze out his cheek. On exam he has right lower lobe crackles and a draining abscess on his mandible. He has very poor dentition. Bronchoscopy with BAL reveals the Gram-positive organism pictured below. What is the likely pathogen? a. Actinomyces b. Mycobacterium tuberculosis c. Streptococcus pneumoniae d. Pseudomonas aeruginosa

A. Thymoma

7. A 55 y.o. female presents with recurrent bacterial sinusitis and pneumonia. CT scan shows bronchiectasis and an anterior mediastinal mass. Additional work-up shows low immunoglobulin levels. What is the most likely diagnosis? A. Thymoma B. Teratoma C. Thyroid Cancer D. Terrible Lymphoma

c. FEV1/FVC ratio: Decreased, FRC: Increased, TLC: Increased, DLCO: Decreased

7. A 78-year-old female presents with shortness of breath. She has smoked two packs of cigarettes per day for 60 years (120 pack years). Chest x-ray is shown. (hyperinflated lungs) What are the most likely findings on PFTs? a. FEV1/FVC ratio: Normal, FRC: Decreased, TLC: Decreased, DLCO: Normal b. FEV1/FVC ratio: Decreased, FRC: Increased, TLC: Normal, DLCO: Normal c. FEV1/FVC ratio: Decreased, FRC: Increased, TLC: Increased, DLCO: Decreased d. FEV1/FVC ratio: Increased, FRC: Decreased, TLC: Decreased, DLCO: Decreased e. FEV1/FVC ratio: Normal, FRC: Normal, TLC: Normal, DLCO: Decreased

c. An improvement in the efficiency of oxygenation Answer C is correct. P/F ratio is used to assess efficiency of oxygenation. Normal P/F ratio is around 500 (normal PaO2 = 80-100, Normal FiO2 = 0.21). ARDS is graded based on the degree of impairment of P/F ratio. Severe ARDS has a P/F ratio < 100, Moderate ARDS has a P/F ratio 100-200, Mild ARDS has a P/F ratio 200-300. An improvement in the P/F ratio represents an improvement in the efficiency of oxygenation. A decline in diffusing capacity or V/Q would represent declining efficiency.

7. A patient with acute respiratory distress syndrome (ARDS) has a change in PaO2/ FiO2 (P/F) ratio from 154 to 237, what does that indicate? a. Worsening ARDS b. A decline in the diffusing capacity of the lung c. An improvement in the efficiency of oxygenation d. A decline in V/Q matching in the lung

b. This patient is at increased risk for lung cancer Answer B is correct. The images show calcified pleural plaques that are thin and smooth. The presence of these lesions are highly suggestive that he has been exposed to asbestos in the past. Calcified pleural plaques usually take decades to appear after exposure. Asbestos exposure is a risk factor for development of lung cancer even in the absence of tobacco abuse. The combination of tobacco smoking and asbestos exposure is synergistic for the risk of developing lung cancer. Asbestosis is an interstitial lung disease related to past exposure to asbestosis. Asbestosis typically presents with shortness of breath, restrictive pulmonary function testing and reticular changes of the lung tissue on CT. Calcified pleural plaques lesions do not transform into malignant mesothelioma. Malignant mesothelioma usually arises from a previously normal area of pleura. Malignant mesothelioma is tightly associated with past asbestos exposure. Alternative causes of calcified pleural plaques are very rare including previous empyema and previous hemothorax, but these are usually unilateral.

7. An 83-year-old steam fitter who served on WW II-era ships while in the Navy presents with the following chest radiograph and no symptoms. Which of the following statements about his prognosis is most correct? (Radiographs show calcified pleural plaques that are thin and smooth.) a. This patient has asbestosis b. This patient is at increased risk for lung cancer c. These lesions commonly transform into malignant mesothelioma d. These lesions are typically unrelated to asbestos exposure.

d. Pulmonary embolism

7. An ABG is obtained from a 48-y.o. female, on room air, at sea level, in the Emergency Department. Which of the following is the most likely etiology of this ABG result? pH = 7.48, PaO2 = 46, PaCO2 = 31, HCO3- = 24, O2 Saturation 88% (Normal values; pH 7.35-7.45, PaO2 80-100, PaCO2 33-45, HCO3- 22-28). Chest X-ray is shown. (Normal chest X-ray) a. Acute respiratory distress syndrome b. Carbon monoxide poisoning c. Opiate overdose d. Pulmonary embolism

c. Aspergillus fumigatus Answer C is correct. This patient likely has Allergic bronchopulmonary aspergillosis (ABPA). ABPA occurs primarily in patients with asthma or cystic fibrosis. It is characterized by recurrent exacerbations with transient pulmonary consolidations and development or worsening of central bronchiectasis. Patients with ABPA often have prominent wheezing on exam. The pulmonary consolidations represent mucus plugging of airways. Patient occasionally cough up the mucus plugs intact. Laboratory abnormalities in ABPA include elevated total blood eosinophil count (> 500 cells/microliter), elevated total serum IgE (often >1000 IU/mL) and serum antibodies against Aspergillus. Skin testing for aspergillus sensitivity is usually the first test of choice during the diagnostic workup. Treatment is with corticosteroids. If the patient fails to improve then antifungal medications can be used. Tuberculosis should be considered in patients with unexplained infiltrates and cough. Fever is the most common symptoms of tuberculosis which this patient does not report. Eosinophilia is also not reported in tuberculosis. Interestingly, the cystic fibrosis gene is postulated to provide protection against tuberculosis (similar to how sickle cell genes protect against malaria). Strongyloides can cause peripheral eosinophilia and transient infiltrates along with bronchoconstriction but is not associated with worsening bronchiectasis. This patient also doesn't have a travel history suggestive of Strongyloides. Pseudomonas is a common colonizer of patients with cystic fibrosis and is associated with increased exacerbations and bronchiectasis; however, it is not associated with peripheral eosinophilia or fleeting infiltrates.

8. 25 y.o. female with cystic fibrosis presents to clinic for evaluation of frequent exacerbation. She is on optimal therapies for cystic fibrosis including elexacaftor, tezacaftor, and ivacaftor. Antibiotics have not been improving her symptoms as they previously did. Upon review of chest imaging she has fleeting pulmonary infiltrates in several areas and worsening central bronchiectasis. Complete blood count reveals an elevated serum eosinophil count of 750 cells/microliter (normal 30-350). This patient's frequent exacerbations are most likely due to colonization of which of the following organisms? a. Mycobacterium tuberculosis b. Strongyloides stercoralis c. Aspergillus fumigatus d. Pseudomonas aeruginosa

d. Dysfunctional dynein arms

8. A 20-year-old female presents to the clinic with recurrent pneumonia. She has frequent sinus and ear infections requiring antibiotics. She has coarse breath sounds on exam. Heart sounds are distant when listening over the normal apex. Chest x-ray reveals situs inversus totalis and bronchiectasis. What is the most likely abnormality in this patient? (organs on opposite side) a. Chloride channel dysfunction b. HIV infection c. Elevated anti-CCP antibody d. Dysfunctional dynein arms

A. Hamartoma

8. A 27 y.o. female presents with an incidental finding on chest CT. She was found to have a 1 cm lung nodule. The nodule contains multiple different densities including fat and calcium. What is the most likely diagnosis? A. Hamartoma B. Lymphoma C. Teratoma

C. Teratoma

8. A 27 y.o. female presents with an incidental finding on chest CT. She was found to have an anterior mediastinal mass. The mass contains multiple different densities including fat and calcium. What is the most likely diagnosis? A. Hamartoma B. Lymphoma C. Teratoma

b. Diabetic ketoacidosis

8. A 53-y.o. male presents with altered mental status and is unable to provide a history. The following blood gas is obtained: pH = 7.28, pO2 = 72, PaCO2 = 32, HCO3- = 16, O2 Saturation 92% (Normal values; pH 7.35-7.45, PaO2 80-100, PaCO2 33-45, HCO3- 22-28) What is the most likely etiology for this blood gas? a. COPD exacerbation b. Diabetic ketoacidosis c. Pulmonary embolism d. Opiate overdose

d. Cryptococcus neoformans Answer D is correct. The picture demonstrates a thick-walled capsule around a yeast that stains clear with India Ink. This is consistent with Cryptococcus. This is an immunosuppressed patient due to her kidney transplant. In addition to presenting as pneumonia, Crytpococcus can also present as meningoencephalitis. Cryptococcus is an opportunistic pathogen in that is typically only affects patient with compromised immune systems. Cryptococcus is present in soil and pigeon droppings and is transmitted via the respiratory system. Aspergillus commonly presents in immunosuppressed patients as well; however, Aspergillus species appear as septate hyphae that branch at acute angles (approximately 45°). Histoplasma is frequently asympomatic in immunocompentent hosts but can become disseminated in the setting of immune suppression. Histologically it appears as a small, intracellular yeast form. Coccidioides can cause pneumonia in immunocompetent patients or disseminated disease in immunosuppressed patients. Coccidioides appears as a large spherule that contains small, round endospores.

8. A 65-year-old female with a history of diabetes mellitus type 2 and kidney transplant presents to the emergency room with fever, cough and pleuritic chest pain. Chest x-ray shows right middle lobe infiltrate with air bronchograms. Bronchoalveolar lavage obtained during bronchoscopy shows the following organisms on India ink stain. What is the most likely pathogen causing the patient's illness? a. Aspergillus fumigatus b. Histoplasma capsulatum c. Coccidioides immitis d. Cryptococcus neoformans

b. 100% oxygen via facemask Answer B is correct. Carbon monoxide poisoning is often difficult to diagnosis because of the falsely normal pulse oximeter and PaO2 readings. Carbon monoxide poisoning can be seen in house fires, attempted suicide via car in garage and broken furnaces. To detect CO poisoning, ABG with co-oximetry and carboxyhemoglobin (CO bound to hemoglobin) levels are required. The SpO2 is falsely high due to carboxyhemoglobin being recognized the same as oxyhemoglobin by pulse oximetry. The PaO2 (the amount of oxygen dissolved in the blood) is usually normal but the amount of oxygen bound to hemoglobin measured by co-oximetry is low. Common symptoms of CO poisoning include headache, nausea and altered mental status. CO has an approximately 240 times increased affinity for hemoglobin than oxygen so displacing it off the hemoglobin molecule is very difficult. For this reason, 100% oxygen is utilized in attempts to displace the carbon monoxide. If coma, seizures or severe neurologic findings are noted then hyperbaric oxygen therapy is used in addition to oxygen therapy. Answer A would be appropriate if cyanide toxicity was a concern. Up to 1/3 of patient in house fire can have cyanide toxicity due to the amount of burned plastics. Cyanide decouples the mitochondria of the cell resulting in purely anaerobic metabolism, thus generative a severe lactic acidosis (i.e. a metabolic acidosis would be noted on ABG). Answer C would be appropriate if a pulmonary embolism was suspected. Blood gas would typically show a respiratory alkalosis and hypoxia. Answer D may be appropriate if the patient is not stabilized with 100% oxygen. High altitude is a known cause of hypoxia with a normal Aa gradient.

8. A 76-year old is pulled from a house fire at Leadville, Colorado. She is mildly confused and has a headache. Her oxygen saturation via pulse oximeter is reading 100% on room air. ABG shows pH 7.35, pCO2 37, paO2 98 (Normal values; pH 7.35-7.45, PaO2 80-100, PaCO2 33-45). In addition to an ABG with co-oximetry to assess carboxyhemoglobin levels, what treatment should be immediately initiated? a. Sodium thiosulfate + Amyl nitrite b. 100% oxygen via facemask c. Heparin infusion d. Descent from altitude

C. Consolidation, increased tactile vocal fremitus, increased whisper pectoriloquy, decreased resonance on percussion Answer C is correct. Noise and vibration are transmitted easier through more solid materials. Consolidations and atelectasis are the most solid the lung can be. The next most solid is normal aerated lung which is why normal lungs have a very fine amount of fremitus and aren't hyper-resonant. Liquid is the next easiest to transmit waves through, followed by air being the most difficult. . Tactile vocal fremitus is tested by asking the patient to repeat a series of words (ninety-nine or one-two-three) and then assessing at multiple locations if the amount of reverberation is felt with the medial edge of the hand. There is decreased vibration felt in the setting of pleural effusion and pneumothorax. There is increased vibration transmitted through a consolidation. Whispered pectoriloquy is tested with the patient whispering a phrase while listening over the area of investigation with a stethoscope. A consolidation transmits whispered sounds more distinctly. Effusion and pneumothorax have decreased sound conductance. Percussion increased resonance over pneumothorax vs dullness to percussion for consolidation or effusion. The correct groupings are listed below. Effusion, Decreased vocal fremitus, whisper pectoriloquy, and resonance on percussion Pneumothorax, Decreased vocal fremitus and whisper pectoriloquy, increased resonance on percussion Consolidation, increased vocal fremitus and whisper pectoriloquy, decreased resonance on percussion

8. A patient presents with decreased breath sounds of the right lung. Which is the correct grouping of physical exam findings? A. Effusion, Increased tactile vocal fremitus, decreased whisper pectoriloquy, decreased resonance on percussion B. Pneumothorax, decreased tactile vocal fremitus, increased whisper pectoriloquy, increased resonance on percussion C. Consolidation, increased tactile vocal fremitus, increased whisper pectoriloquy, decreased resonance on percussion D. Pneumothorax, increased tactile vocal fremitus, decreased whisper pectoriloquy, increased resonance on percussion E. Consolidation, decreased tactile vocal fremitus, increased whisper pectoriloquy, decreased resonance on percussion

a. They indicate an increased risk of lung cancer

A 57-yr old former miner from Montana presents for referral following an abnormal chest radiograph. He states he has no dyspnea. Chest radiograph is shown. (Radiographs show calcified pleural plaques that are thin and smooth.) Which of the following statements about the radiographic findings is true? a. They indicate an increased risk of lung cancer b. They indicate that this patient has malignant mesothelioma c. They indicate that this patient has asbestosis

a. Acute eosinophilic pneumonia Answer A is correct. Acute eosinophilic pneumonia presents like a typical bacterial pneumonia with fevers, chills, and productive cough over a matter of days. The classic presentation occurs around the time of a new exposure (increased or new smoking, starting marijuana, or initiation of a new medication such as an antibiotics). Usually presents in young (20-40 y.o.) males. There is frequently bilateral infiltrates and bilateral pleural effusion on chest imaging. The diagnosis is made by bronchoalvelolar lavage showing a high eosinophil count. In acute eosinophilic pneumonia, there is rarely a peripheral eosinophilia (there are no increased eosinophils on CBC from "peripheral" blood). Parasitic (strongyloides) and fungal (coccidomycosis) infections must be ruled out. Chronic eosinophilic pneumonia is not associated with a new exposure and has longer time course then acute eosinophilic pneumonia. Chronic eosinophilic pneumonia frequently occurs in non-smokers. Radiographically it is characterized by migratory peripheral/pleural based infiltrates and is sometimes referred to as "Photonegative of pulmonary edema." (Pulmonary edema usually occurs centrally so the "photonegative" is the opposite). In 85% of chronic eosinophilic pneumonia cases there is both bronchoalveolar and peripheral eosinophilia. Allergic bronchopulmonary aspergillosis presents with central bronchiectasis in a patient with either asthma or cystic fibrosis. Eosinophilic granulomatosis with polyangiitis (previously Churg-Strauss) occurs in patients with severe, uncontrolled asthma and presents with migratory pulmonary infiltrates and peripheral eosinophilia. During the vasculitic phase of illness, there can be skin and renal involvement and is associated with ANCA (Anti-Neutrophil Cytoplasmic Antibodies) positivity.

A 16-yr-old student was given Minocycline for severe acne two weeks ago and presents with fever and dyspnea. Chest radiograph shows bilateral diffuse infiltrates and bronchoalveolar lavage from bronchoscopy shows 32% eosinophils and no organisms. Complete blood cell count shows a normal differential. What is the most likely diagnosis? a. Acute eosinophilic pneumonia b. Chronic eosinophilic pneumonia c. Allergic bronchopulmonary aspergillosis d. Eosinophilic granulomatosis with polyangiitis

A. Recent travel to Southeast California Answer A is correct. Coccidioides immitis is endemic in the Southwest United States. Coccidioides immitis is also known as San Joaquin Valley fever (San Joaquin Valley is located in South-central California) or Coccidiomycosis. Coccidioides is a dimorphic yeast. Dimorphic means it is a round yeast when at body temperature and forms mold hyphae at lower temperature. Large spherules filled with endospores (as shown below) are the most common morphologic form of Coccidioides in clinical specimens. These spherules are not easily visualized on Gram stain but can be visualized with Papanicolau, hematoxylin and eosin (H&E), periodic acid Schiff (PAS) or Gomori-methanamine silver (GMS) stains. In immunocompetent hosts, C. immitis can cause a flu-like illness and pneumonia as described in this patient. In immunocompromised hosts, C. immitis can become disseminated and cause bone, skin and joint lesions and meningitis. Cave exploration in Ohio would put a person at risk for Histoplasma capsulatum and Blastomyces dermatitidis. Histoplasma and Blastomyces are similar to Coccidioides in that they are dimorphic yeast. Histoplasma capsulatum is found in the Missouri and Ohio River basins and is found in bird and bat droppings. Histoplasma is a small, intracellular yeast. Histoplasma is frequently asymptomatic in immunocompetent hosts but can become disseminated in the setting of immune suppression. Blastomycosis is endemic in organic materials east of the Mississippi. It is carried by many animals like dogs and horses. Blastomyces is a large yeast with characteristic "broad-based budding". Narrow based (example Candida sp.) Symptoms of blastomycosis include flu-like illness (fevers, chills, myalgias, and headache) or pneumonia. In immunocompromised hosts, blastomycosis can cause disseminated disease with fevers, night sweats, weight loss, cough, skin rash with pustules or ulcers, meningitis and lytic bone lesions. HIV infection and exposure to pigeon dropping is suggestive of Cryptococcus neoformans. Cryptococcus is present in pigeon droppings. It can cause pneumonia and meningoencephalitis in immunocompromised patients. Cryptococcus is an opportunist pathogen in that typically only affects patient with compromised immune systems. Paracoccidioidomycosis is very similar clinical presentation to coccidioidomycosis but has a different geographic (Rural Latin America) and histological ("Captain's Wheel" appearance, below) profile.

A 35-year-old female presents to your clinic with 3 days of fevers, headache, myalgias and sharp chest pain that increases with inspiration. On exam she has a fever, tachycardia and mild crackles over the left lower lobe. Chest x-ray shows infiltrate of the left lower lobe. She is given empiric antibiotics but returns 4 days later with persistent symptoms. Bronchoscopy is performed and large spherules filled with endospores are noted on Gomori-methenamine silver stain. What is a detailed history likely to find? A. Recent travel to Southeast California B. Cave exploration in Ohio C. HIV infection and exposure to pigeon droppings D. History of living in the Brazilian rainforest

a. Empyema

A 50-year-old male presents with shortness of breath. He has associated fevers, chills and cough. Chest ultrasound reveals a septated right-sided pleural effusion with an associated consolidation. Ultrasound guided thoracentesis is performed and reveals purulent drainage. Gram stain is negative for bacteria. Cell count reveals a neutrophil predominance. Triglycerides are 45 mg/dL. Cytology is negative. What is the most likely diagnosis? a. Empyema b. Chylothorax c. Malignant pleural effusion d. Simple parapneumonic effusion e. Complex parapneumonic effusion

c. Fat embolism Answer C is correct. Fat emboli occur in the setting of long bone or pelvic fractures, which release bone marrow into the systemic circulation. The onset of symptoms is delayed from 24-72 hours after fracture. The delay is due to the immunogenic response against circulating antigens, not just the fat moving about the circulation. Symptoms include respiratory distress, hypoxia, bilateral infiltrates that mimic ARDS, altered mental status, seizures, petechial rash preferentially on the chest and axilla. Occasionally, fat emboli syndrome can result in coagulopathy and thrombocytopenia. Pulmonary embolism is possible in the setting of immobility from a fracture; however, bilateral infiltrates on chest x-ray make pulmonary embolism less likely as PE normally presents with clear chest imagining. Air embolism most commonly occurs in the setting of interventions on the vascular system (example: central venous catheter insertion). If a bolus of venous air is large enough it can occlude the right ventricular outflow tract and cause cardiovascular collapse. Amniotic fluid embolism occurs when amniotic fluid enters the maternal circulation. Luckily, this is a rare complication of delivery. Patients typically present with rapidly progressive hypoxic respiratory failure and cardiovascular collapse followed by hemorrhage for disseminated intravascular coagulation. As this patient is not known to be pregnant, amniotic fluid embolism is unlikely.

A 25 yr-old female is hit by a SUV while riding her bicycle and suffers a right femoral fracture but no other injuries. Two days later, while recuperating in hospital following orthopedic surgery, she develops progressive respiratory distress and refractory hypoxemia. Chest radiograph shows patchy haziness in both lungs. Which of the following is the most likely cause of her hypoxemia? a. Air embolism b. Thromboembolism c. Fat embolism d. Amniotic fluid embolism

d. Influenza virus Answer D is correct. Influenza virus aka "The Flu" is an orthomyxovirus. It causes a clinic syndrome of abrupt onset fever, headache, myalgia, and malaise. Symptoms usually improve over 5-7 days. Outbreaks of seasonal influenza frequently occur. Haemophilus influenza is a bacterial pneumonia. Haemophilus influenza (H.flu) is a "typical" bacteria pathogen. The terms "typical" and "atypical" pneumonia were coined based on their response to penicillin-based antibiotics. Generally typical pneumonia response to therapy and are thought of as lobar (one lobe of the lung) consolidations and classic symptoms of pneumonia including fever, productive cough and shortness of breath. "Atypical" pneumonia classically failed penicillin and are often called "Walking" pneumonia due to its less severe presentation. It often presents with less dense but bilateral consolidation on chest imaging. H. flu requires chocolate agar with factor V (NAD) and X (hematin) to grow. It is an oxidase-positive Gram-negative rod. H.flu is a common cause of otitis media (along with Streptococcus pneumonia). H.flu Type B is encapsulated by a polysaccharide capsule that prevents phagocytosis. Because of this polysaccharide capsule a Haemophilus influenzae Type b (Hib) vaccine is available. However, Hib accounts for only 10% of H.flu related acute otitis media in children with the rest resulting from non-typable (non-encepasulate) H.flu. The utility of Hib vaccination comes from its ability to prevent Hib associated meningitis. Respiratory syncytial virus (RSV) is a paramyxovirus that typically present with both upper and lower respiratory involvement. Sinusitis with rhinorrhea (runny nose), conjunctivitis (pink eye), bronchitis and pneumonia can occur. Myalgias can occur but are not usually as severe as described here. Also the lack of upper respiratory symptoms make RSV less likely than Influenza. Rotavirus is a common cause of viral gastroenteritis sometimes call the stomach flu. Symptoms of rotavirus include watery diarrhea and vomiting.

A 25-year-old medical student develops fever, myalgias and a non-productive cough. The myalgias are so severe she says "I feel like I got hit by a truck." She misses class for a few days but the myalgias and fevers resolved. Severe other members of her class have a similar illness. A chest radiograph is shown with bilateral consolidation. Which of the following is the most likely pathogen to be responsible for the patient's illness? a. Haemophilus influenza b. Rotavirus c. Respiratory syncytial virus d. Influenza virus

c. Primary spontaneous pneumothorax

A 25-year-old tall male presents to the Emergency Department with severe, pleuritic, right upper chest pain. He has no known lung diseases. Chest X-ray shows a small pneumothorax without a shift in the mediastinum. Which type of pneumothorax is this? a. Tension pneumothorax b. Secondary spontaneous pneumothorax c. Primary spontaneous pneumothorax d. Iatrogenic pneumonthorax

a. Reduced expiratory reserve volume Answer A is correct. In both obesity and pregnancy, the expiratory reserve volume is reduced. This is due to the abdominal contents pushing the diaphragm up. Restrictive lung diseases have a preserve/normal or increased FEV1/FVC ratio. Obstructive lung diseases like asthma and COPD have a reduced FEV1/FVC ratio. Functional residual capacity is the sum of expiratory reserve volume and residual volume so when expiratory reserve volume goes down, the functional residual capacity goes down as well.

A 28-yr-old female presents with shortness of breath over the last one month. She is 36 weeks pregnant. She has never had shortness of breath before. Her past medical history is significant for an uneventful full term delivery 4 years prior. On exam, she has no wheezing, no ankle swelling and a gravid uterus consistent with gestational age. What is the most likely finding on PFTs? a. Reduced expiratory reserve volume b. Increased functional residual capacity c. Reduced FEV1/FVC ratio d. Increased vital capacity

a. Hemothorax

A 33-year-old patient presents after head-on motor vehicle accident. Patient is complaining of chest pain and shortness of breath. He is diaphoretic and tachycardic. Blood pressure and pulse oximetry are normal. On physical exam he has bruising over his right side. Anterior-posterior compression of his rib cage results in severe pain of the right lateral chest over an area of ecchymosis. There is dullness to percussion over the right base. What is most likely responsible for his physical exam findings? a. Hemothorax b. Pneumothorax c. Chylothorax

A. Lymphangioleiomyomatosis Answer A is correct. Lymphangioleiomyomatosis (LAM) is a cystic lung disease that presents almost exclusively in women. The cysts are prone to rupture and cause pneumothorax. These patients are also at risk for chylothorax. On biopsy LAM shows immature smooth muscle and spindle cells along lymphatics. Pulmonary Langerhans Cell Histiocytosis (PLCH) is typically seen in smokers. Chest imaging findings include multiple thin walled cysts and centrilobular ground glass nodules. Sarcodosis can present in young females but cystic lung disease and pneumothorax in sarcoid are rare. Radiographic findings in sarcoid are typically mediastinal and hilar lymphadenopathy. Desquamative interstitial pneumonia (DIP) is a smoking related ILD that radiographically presents with diffuse ground glass opacities bilaterally.

A 33-yr-old female presents with acute onset shortness of breath. CT scan shows pneumothorax and multiple thin walled cysts. What are the most likely pathologic findings? A. Lymphangioleiomyomatosis B. Pulmonary Langerhans Cell Histiocytosis C. Sarcoidosis D. Desquamative Interstitial Pneumonia

b. Begin anticoagulation for 3 months Answer B is correct. The patient has a provoked DVT and 3 months of therapeutic anticoagulation is recommended. In this case a novel oral anticoagulant like a Factor Xa inhibitor would be considered first line therapy. The patient's DVT was provoked by immobility secondary to his leg fracture. This is the patient's first DVT and hypercoagulability work-up is not recommended. Only after two or more unprovoked thrombovenoembolisms is further testing recommended. D-dimer is useful to rule out DVT or PE in the setting of low pre-test probability based on the Wells Criteria. Antiplatelet therapy with aspirin has no role in treatment of an acute DVT.

A 34 y.o. male presents to clinic for evaluation of a swollen, increasingly painful, right leg. He broke his right ankle 4 weeks ago and has been in a cast since then. His pain had previously been controlled but he has developed a new type of pressure in his leg. Lower extremity Doppler ultrasound reveals a deep vein thrombosis in the right common femoral vein. The patient denies shortness of breath. He has no other medical problems. Which of the following is the most appropriate next step in management? a. Check a D-dimer b. Begin anticoagulation for 3 months c. Begin aspirin indefinitely d. Begin hypercoagulability work-up

d. Anaerobic Gram-positive cocci and Gram-negative rods Answer D is correct. The chest CT shows a left lower lobe cavitary lesion. Based on his history of foul-smelling breath, fever and productive cough, this likely represents an aspiration pneumonia. One of the most common causes of cavitary pneumonias is aspiration of oral contents. Alcoholics are at increased risk of developing several types of pneumonia and they are at a particularly high risk of aspiration pneumonia. Aspiration pneumonia occurs when there is impairment of the gag and cough reflexes and oral flora are aspirated into the lung. Other common risk factors for aspiration pneumonia include drug abuse, seizure disorders, stroke and dementia. Poor dentition is commonly seen in these patient populations and is associated with higher bacterial counts in the mouth. Oral anaerobic bacteria include Bacteroides (Gram-positive), Prevotella (Gram-negative), Fusobacterium (Gram-negative) and Peptostreptococcus (Gram-positive). Anaerobes are notorious for their foul smell. Gram-negative rods that commonly cause pneumonia include Pseudomonas aeruginosa, Haemophilus influenzae, Klebsiella pneumoniae and Legionella pneumophila. None of these organisms typically cause cavitary pneumonia. Acid-fast bacilli would include Mycobacterium infections from both tuberculosis and non-tuberculous Mycobacterium. Tuberculosis can cause lung cavities but this patient has no listed risk factors for Tb. Gram-positive diplococci would be indicative of Streptococcus pneumoniae, which does not typically cause cavitary pneumonia.

A 35-year-old lawyer comes into the emergency room with symptoms of fever and productive cough. He admits to frequent binge drinking and often passes out at the end of the night. He has poor dentition and foul-smelling breath. CXR shows left lower lobe air fluid level and CT confirms left lower lobe lung abscess. Which of the following is the most likely to be found on his sputum Gram stain and culture? a. Aerobic Gram-negative rods b. Acid-fast bacilli c. Aerobic Gram-positive diplococci d. Anaerobic Gram-positive cocci and Gram-negative rods

c. Sarcoidosis Answer C is correct. Sarcoidosis is the most likely etiology with classic findings of bilateral hilar lymphadenopathy and non-caseating granulomas. Tuberculosis is unlikely based on the lack of organisms seen and tuberculosis causes caseating (necrotizing) granulomas. Blastomycosis is an endemic fungal infection and would typically be seen in the bronchoalveolar lavage specimen. Berylliosis and Sarcoidosis both cause non-caseating granulomas. Beryllium exposure occurs in manufacturing and processing of alloys. Her occupation does not put her at risk.

A 35-yr-old female office worker presents with bilateral hilar adenopathy on chest radiograph but no symptoms of respiratory difficulty. She undergoes bronchoscopy with biopsy shows non-caseating granulomas and gram stain and culture from bronchoalveolar lavage (aka lung washings) show no organisms. What is the most likely cause of this pathology? a. Blastomycosis b. Tuberculosis c. Sarcoidosis d. Berylliosis

d. Plugs of granulation tissue in the distal airways with surrounding cellular inflammation

A 36-yr-old male presents with 1 month of productive cough, fever and shortness of breath. He has been seen by his primary care physician several times in the last month. At the time of symptoms onset he was found to have a right upper lobe infiltrate concerning for pneumonia and prescribed appropriate community acquired pneumonia antibiotics. His symptoms did not improved so his antibiotics where changed to included coverage for resistant organisms. His symptoms again failed to improve and a repeat chest x-ray now reveals a left lower lobe infiltrate. He was sent for lung biopsy. What are the most likely pathologic findings? a. Eosinophilic granulomas and diffuse cysts in the upper lobes b. Noncaseating accumulation of activated macrophages c. Abundant tan pigmented macrophages within the alveolus d. Plugs of granulation tissue in the distal airways with surrounding cellular inflammation

c. Chronic eosinophilic pneumonia

A 37 yr-old male presents with dyspnea on exertion, fever, weight loss and cough that has been productive over the past 2 months. Chest radiograph shows peripheral and pleural-based haziness that is described as the 'radiographic negative of pulmonary edema.' A CBC reveals an elevated eosinophil count and bronchoscopy with lavage was done demonstrating an eosinophil count of 45%. What is the most likely diagnosis? a. Acute eosinophilic pneumonia b. Allergic bronchopulmonary aspergillosis c. Chronic eosinophilic pneumonia d. Eosinophilic granulomatosis with polyangiitis (Churg-Strauss syndrome)

c. Pneumocystis jirovecii Answer C is correct. Pneumocystis jirovecii is the causative pathogen in Pneumocystis Pneumonia (PCP) or Pneumocystis jirovecii pneumonia (PJP). Pneumocystis is currently classified as a fungus. PCP presents with gradual onset of fevers, cough and shortness of breath. Patients with HIV and a CD4 count < 200 cells/microL are at particular risk of developing this opportunistic infection and prophylactic antibiotics are thus recommended. High LDH is common in PCP. Another blood test that can be used to evaluate for PCP is 1-3-beta-D-glucan (aka Fungitell). 1-3-beta-D-glucan is a component of the cell wall of several fungi and is therefore not specific for PCP. Chest imaging will commonly shows diffuse, bilateral infiltrates but chest x-rays can be normal in up to 25% of patients with PCP. Patients with PCP often become severely hypoxic during the treatment course due to the massive amount of inflammation. Bronchoalveolar lavage obtained during bronchoscopy will typically show Pneumocystis jirovecii organisms on silver stain. Below you can appreciate the "dented ping-pong ball" appearance. Patients with HIV are at increased risk for active Mycobacterium tuberculosis (Tb) infection as well. Mycobacterium would typically stain positive with acid-fast staining (aka Ziehl-Neelsen). Patients with HIV are monitored for latent Tb. They have different cut-offs for their purified protein derivative (PPD) skin testing. A PPD is considered positive if > 15 mm of induration in healthy, non-at-risk individuals. Due to the high risk of conversion to active Tb in HIV patients, a PPD > 5 mm is considered positive. Mycobacterium avium complex (MAC) should also be considered in the differential diagnosis. Patients with HIV are typically at risk for MAC when their CD4 count drops to < 50 cells/microL but MAC typically causes disseminated disease rather than pneumonia. And again, Mycobacterium would typically stain positive with acid-fast staining. Streptococcus pneumoniae is the most common cause of pneumonia in HIV patients; however, the presence of a high LDH and cystic organisms on silver stain make PCP a more likely etiology.

A 40-year-old male with HIV positive presents with increased shortness of breath, fevers and cough over the last one month. His last know CD4 count was 100 cells/microL. He does not take antiretroviral medications. He is a smoker. On exam he has an oxygen saturation of 89% and bilateral crackles. His laboratory reveals a lymphopenia and an LDH of 400 IU (Normal: 100-190 IU). Chest x-ray shows bilateral interstitial infiltrates. Bronchoscopy is performed. Acid-fast and Gram staining are negative. Silver stain reveals multiple small round cystic elements. What is the most likely pathogen causing his symptoms? a. Mycobacterium tuberculosis b. Mycobacterium avium complex c. Pneumocystis jirovecii d. Streptococcus pneumoniae

b. Preserved lung architecture with alveoli full of amorphous material that stains positive for PAS Answer B is correct. The patient has Pulmonary alveolar proteinosis (PAP). PAP commonly presents in young male smokers with non-specific respiratory symptoms and a restrictive lung disease on PFTs. The pattern on CT with intralobular thickening and diffuse ground-glass opacities is sometimes referred to as "Crazy Paving". PAP is one of the only diseases where milky BAL fluid is seen. The amorphous material seen is surfactant protein. Answer A describes lymphangioleiomyomatosis (LAM). LAM is typically seen in young women. CT findings of LAM include multiple cysts. Pathologically there is smooth muscle with in the lung along the lymphatics. Answer C describes Sarcoidosis with non-necrotizing granulomas. Sarcoid can present in young males with restrictive lung disease. The absence of lymphadenopathy and a milky BAL would be uncommon in Sarcoidosis. Answer D describes emphysema. This would present with obstructive PFTs with a reduced FEV1/FVC ratio and a CT scan would frequently show emphysematous blebs and hyperinflation. Answer E describes Pulmonary Langerhans Cell Histiocytosis, aka Eosinophilic granuloma. This disease would present with upper lobe predominant nodules and cysts. Due to the cysts formation, pneumothorax is common. PLCH is usually seen in smokers.

A 40-yr-old male present with increasing shortness of breath and cough. Patient is an active smoker. He has no past medical history. Pulmonary function testing reveals a restrictive pattern with FEV1/FVC 90%, FVC 70%, TLC 73% and DLCO 50%. CT scan reveals intralobular thickening and diffuse ground-glass opacities and no mediastinal lymphadenopathy. Bronchoscopy with aleolar lavage is performed and reveals a milky fluid. What pattern is most likely to be seen on lung biopsy? a. Cysts with immature smooth muscle and spindle cells along lymphatics b. Preserved lung architecture with alveoli full of amorphous material that stains positive for PAS c. Accumulation of activated epithelioid histiocytes without necrosis and occasional multinucleated giant cells d. Abnormal enlargement of alveoli with destruction of the alveolar septa without fibrosis e. Eosinophilic granulomas with large, ovoid phagocytes with eosinophilic cytoplasm, prominently grooved nucleus and diffuse cysts

b. Hypersensitivity pneumonitis Answer B is correct. Hypersensitivity pneumonitis (HP) is characterized by the body's overreaction to inhaled organic antigens. The classic presentation is recurrent flu like illness with each exposure to the inhaled antigen. HP typically occurs in the bilateral upper lobes (where the particles go). The body forms loosely formed granulomas around the antigens. There are thousands of identified antigens that cause HP including bird feathers, mold from flooding exposure, silo dust, and mycobacteria that can be found in hot tubs. Smokers rarely develop HP because the body is busy fighting off tobacco byproducts so it does not have time to develop a hypersensitivity reaction. Idiopathic pulmonary fibrosis (IPF) typically presents in older populations (usually > 50 y.o.). IPF also presents with chronic (years) of symptoms. The diagnosis is often made based on characteristic radiographic findings of bilateral (both sides), lower lung predominant peripheral based honeycombing and traction bronchiectasis in the absence of other identifiable causes of lung scarring. Acute eosinophilic pneumonia presents like a typical bacterial pneumonia with fevers, chills, and productive cough over a matter of days. There is frequently bilateral infiltrates on chest imaging but it does not have a preference for the upper lobe. The diagnosis is made by bronchoalvelolar lavage showing a high eosinophil count. Desquamative interstitial pneumonia is diagnosed in smokers and has a similar radiographic presentation to HP.

A 45-yr-old non-smoking male presents with productive cough and worsening dyspnea. He has not had any occupational exposures. He did inherit two cockatiels from his uncle when he died 2 months ago. He has bilateral patchy upper lobe infiltrates on his chest radiograph. Pulmonary function testing reveals an FEV1 60% predicted, FVC 60% predicted and FEV1/FVC 100%. What is the most likely diagnosis? a. Idiopathic pulmonary fibrosis b. Hypersensitivity pneumonitis c. Acute eosinophilic pneumonia d. Desquamative interstitial pneumonia

a. Desquamative interstitial pneumonia

A 48 yr-old smoker presents with dyspnea on exertion and pulmonary function tests reveal a mild reduction in TLC and RV with a moderate decrease in DLCO. A CT image is shown. If the lung findings on CT were to be biopsied, they would show relatively normal alveolar architecture and alveoli packed with alveolar macrophages. (Don't necessarily need CT. Think SMOKER and ALVEOLAR MACROPHAGES) Which of the following is the most likely diagnosis? a. Desquamative interstitial pneumonia b. Non-specific interstitial pneumonia c. Usual interstitial pneumonia d. Hypersensitivity pneumonitis

d. Begin treatment with prednisone

A 50-yr old female presents with dyspnea on exertion and a cough. She works in an insurance office and has had no industrial exposures. Her chest radiograph is shown. Interferon Gamma Releasing Assay is negative. Bronchoscopy is done and biopsies of lung tissue demonstrate tightly formed non-caseating granulomas with no evidence of fungal or yeast forms. Acid-fast testing is negative. (Chest radiograph shows bilateral hilar lymphadenopathy) What is next the best step in the management of this patient? a. Beryllium proliferation test of BAL and serum b. Advise avoidance of known antigens c. Begin treatment with isoniazid, rifampin, pyrazinamide and ethambutol d. Begin treatment with prednisone

b. Non-specific interstitial pneumonia

A 53-yr-old non-smoker presents with dyspnea on exertion. Past medical history is significant for Polymyositis. Pulmonary function tests that indicate a restrictive lung disease as evidence by normal FEV1/FVC ratio with reduced FVC and TLC. CT chest reveals multiple areas of ground glass opacities without honeycombing. The patient is given prednisone 1mg/kg daily for 3 months and on return to clinic states his symptoms are much improved. Which of the following is the most likely diagnosis? a. Desquamative interstitial pneumonia b. Non-specific interstitial pneumonia c. Pulmonary Langerhans Cell Histiocytosis d. Usual interstitial pneumonia

b. Pleural LDH/Serum LDH greater than 0.6

A 56-year-old male presents with a pneumonia and pleural effusion and a thoracentesis is done. Laboratory results are as follows: WBC 14 (Normal 6-12), Serum LDH 250 U/L (Normal 98-192), Serum Protein 4.5 g/dL (5.9-7.5), and thoracentesis results in 200 cc of amber fluid with the following characteristics: pH 7.35, LDH 220 U/L, protein 2.0 g/dL. According to Light's criteria, which of the following criteria would characterize this as an exudative effusion? a. WBC greater than 12 b. Pleural LDH/Serum LDH greater than 0.6 c. Pleural Protein/Serum Protein less than 0.5 d. Pleural fluid pH less than 7.40

b. Klebsiella pneumoniae Answer B is correct. Klebsiella pneumoniae is an encapsulated, lactose fermenting, Gram-negative rod. This picture shows the Gram-negative (red) rods with a capsule around. Gram-negative bacteria are uncommon pathogens in healthy patients with community acquired pneumonia; however, alcoholics and diabetics are at increased risk of developing pneumonia from these including Klebsiella. The sputum from patient's infected with Klebsiella are often described as "Red currant jelly." Red currant is a fruit pictured below for reference. Chlamydia is not typically seen on Gram stain. It is difficult to stain due to the fact that it is an intracellular pathogen but is classified as a Gram-negative. Chlamydia is an atypical pneumonia and typically presents with mild symptoms and diffuse (rather than lobar) infiltrate on chest x-ray. Legionella is intracellular weakly Gram-negative rod. It frequently is transmitted via aerosolization of a contaminated water source. Legionella is an "atypical" pathogen in that it does not respond to penicillin based antibiotics, but behaves similar to "typical" pathogens in the lobar nature and more severe presentation. Legionella is frequently associated with hyponatremia (low sodium) and diarrhea. Streptococcus pneumoniae appears as Gram-positive cocci, often in pairs (diplococci) on Gram stain. It classically is associated with rust colored sputum production.

A 57-year-old homeless man with diabetes presents with cough productive of thick, reddish sputum with clots, fever and dyspnea. CXR shows right middle lobe consolidation. Sputum Gram stain is shown. The picture shows the Gram-negative (red) rods with a capsule around them. What is the most likely organism causing this pneumonia? a. Chlamydia pneumoniae b. Klebsiella pneumoniae c. Legionella pneumophila d. Streptococcus pneumoniae

d. Pseudomonas aeruginosa Answer D is correct. Ventilator and Hospital acquired pneumonia has a different microbiology that is typical for community acquired pneumonia. Pseudomonas aeruginosa and Staphylococcus aureus are much more prevalent in hospitalized patients and in those requiring ventilatory support. Pseudomonas aeruginosa is an aerobic non-lactose fermenting, oxidase positive, Gram-negative rod. Pseudomonas organisms produces pyocyanin which has a characteristic blue-green pigment. Staphylococcus aureus is a Gram-positive cocci that occurs in clusters. Streptococcus pneumonia is the most common bacterial pathogen isolated in community acquired pneumonia and is a Gram-positive diplococci. Mycoplasma pneumonia presents as an "atypical" community acquired pneumonia and is not seen on Gram stain due to its small size and lack of cell wall.

A 60-year-old female is admitted with a stroke resulting in facial droop and inability to move her right side. She is intubated due to decreasing level of consciousness. Five days later she develops crackles over her left lung, increased secretions that are green in color and worsening oxygenation. CXR confirms a lingular pneumonia. Sputum Gram stain reveals a Gram-negative rod. What is the most likely pathogen responsible for this patient's ventilator associated pneumonia? a. Streptococcus pneumonia b. Mycoplasma pneumonia c. Staphylococcus aureus d. Pseudomonas aeruginosa

b. FEV1: 70%. FVC: 71%. ERV: 70%. RV: 80%. TLC: 75%.

A 65 y.o. female was admitted with pancreatitis and then develops severe Acute Respiratory Distress Syndrome (ARDS) as evidence by bilateral interstitial infiltrates and severe hypoxia with a P/F ratio of 70. Patient received mechanical ventilation and maximal supportive cares. Patient recovers from her acute illness but has mild residual shortness of breath. Prior to discharge from the hospital she undergoes pulmonary function testing. What is the most likely pattern seen on PFTs? a. FEV1: 65%. FVC: 92%. ERV: 98%. RV: 130%. TLC: 105%. b. FEV1: 70%. FVC: 71%. ERV: 70%. RV: 80%. TLC: 75%. c. FEV1: 89%. FVC: 93%. ERV: 95%. RV: 85%. TLC: 91%. d. FEV1: 88%. FVC 87%. ERV: 50%. RV: 86%. TLC: 83%

b. Nocardia asteroides Answer B is correct. Nocardia is an opportunistic infection and this patient's kidney transplant puts her at risk. Nocardia is unique in that it is a Gram-positive, branching filamentous rod. Despite being a bacteria, Nocardia is weakly acid-fast. It is endemic in soil (it is found everywhere). In immunocompromised hosts, Nocardia can cause lung, skin and brain abscesses so the combination of both pulmonary nodules and a ring-enhancing lesion on MRI points to Nocardia. Candida is an extremely rare cause of pneumonia, even in immunosuppressed patients. Histologically, Candida appears as pseudohyphae and budding yeast. KOH preps and silver stains allow the visualization of Candida more clearly. Streptococcus pneumoniae is the most common bacterial pathogen identified in a wide range of populations. On Gram stain it appears as a Gram-positive diplocci. Cryptococcus is another opportunistic infection and can cause both pneumonia and meningitis; however, it doesn't cause ring enhancing lesions. Histologically, it has a thick capsule around a round yeast.

A 65-year-old female with history of diabetes mellitus type 2 and kidney transplant presents to the emergency room with fever, night sweats, cough and headache. On exam she is febrile. Chest x-ray shows bilateral nodules. Brain MRI shows a ring-enhancing lesion. Sputum Gram stain is shown. Gram-positive, branching filamentous rod. Which organism is most responsible for this patient's symptoms? a. Candida albicans b. Nocardia asteroides c. Streptococcus pneumoniae d. Cryptococcus neoformans

Coal worker pneumoconiosis

A 65-yr-old male presents for a routine physical. He just moved here from West Virginia to be closer to his grandkids. He is a lifelong smoker with a 30-pack year history. His brother had Sarcoidosis. Patient has held several jobs in his career including auto mechanic, welder and coal miner. Chest x-ray show numerous upper lobe nodules all less than 1 cm in size. What is the most likely diagnosis? Asbestosis Coal worker pneumoconiosis Respiratory Bronchiolitis associated Interstitial Lung Disease Sarcoidosis COPD

a. Legionella pneumophila Answer A is correct. The chest x-ray shows a right upper lobe pneumonia. The patient has hyponatremia (low sodium) and diarrhea associated with this pneumonia. Those findings along with recent cruise should point you toward Legionella as the likely culprit. Legionella is difficult to culture and requires charcoal yeast extract agar buffered with increased iron and L-cysteine. Legionella is a facultative intracellular weakly Gram-negative rod. Legionella can be seen better with a silver stain which is usually used to identify fungus. Legionella outbreaks are common due to its aerosol transmission usually from a contaminated water source. Cruise ships, hotel water displays, nursing home and recently in the news hospitals have been the implicated sources. Patient with Legionella pneumonia (also known as Legionnaires' disease) also have a relative bradycardia in the setting of high fever (> 39 C). Chlamydia is not typically seen on Gram stain. It is difficult to stain due to the fact that it is an intracellular pathogen but is classified as a Gram-negative. Chlamydia is an atypical pneumonia and typically presents with mild symptoms and diffuse (rather than lobar) infiltrate on chest x-ray. Mycoplasma pneumonia presents as an "atypical" community acquired pneumonia and is not seen on Gram stain due to its small size and lack of cell wall. Streptococcus pneumoniae is the most common bacterial pathogen isolated in community acquired pneumonia and is a Gram-positive diplococci. Streptococcus pneumoniae is a "typical" pneumonia.

A 67-year-old female develops myalgias, diarrhea, fever and a headache shortly after returning from a cruise in the Mediterranean and presents to the Emergency Department with cough and dyspnea. Gram stain of sputum demonstrated bacteria but they were difficult to identify and the sputum has been cultured. Labs are obtained and reveal as sodium of 120 mEq/L (Normal: 135-145 mEq/L). Chest radiograph is shown: The chest x-ray shows a right upper lobe pneumonia. What is the most likely organism causing this pneumonia? a. Legionella pneumophila b. Chlamydia pneumoniae c. Mycoplasma pneumoniae d. Streptococcus pneumoniae

c. Increased hydrostatic pressure

A 67-year-old male presents to the emergency department with shortness of breath that has been increasing over the past few days. An upright chest radiograph demonstrates an enlarged heart, more visible pulmonary vessels in the upper lung fields and diffuse alveolar infiltrate suggestive of pulmonary edema along with bilateral pleural effusions. What is the most likely mechanism of these effusions? a. Decreased lymphatic drainage b. Decreased osmotic pressure c. Increased hydrostatic pressure d. Increased vascular permeability

d. Mesothelioma Answer D is correct. This patient with asbestos exposure presenting with thick, non-calcified pleural plaques likely has Mesothelioma. Asbestos exposure is the only known risk factor for development of mesothelioma. Radiographically one should be suspicious of mesothelioma if the pleura plaque has nodularity (lumpy-bumpy), thickness > 1 cm or involvement of mediastinal pleura. Benign asbestos related pleural plaques are very different. They typically calcify over time, are smooth, are < 1 cm and do not involve the mediastinal pleura. Asbestos related pleural plaques are not pre-malignant. They do not transform into mesothelioma. They are a marker for previous asbestos exposure. Asbestos exposure increases your risk of both mesothelioma and lung/bronchogenic carcinoma. Lung cancer typically presents with a nodule or mass within the lung tissue, not within the pleura. Asbestosis is an interstitial lung disease with restrictive PFTs and bilateral reticular changes on CT.

A 67-yr-old male HVAC worker presents with an abnormal chest CT. His CT shows multiple non-calcified areas of pleural thickening many of which are greater than 2 cm thick. He states that he has been exposed to asbestos many times during his employment. What is most likely responsible for the pleural thickening? a. Asbestosis b. Pleural plaques c. Lung adenocarcinoma d. Mesothelioma

b. Tuberculosis Answer B is correct. Silicosis typically develops in patients exposed to mining, sand blasting, stone cutting, masonry, glass and ceramic manufacturing. Typical or simple silicosis develops 10-30 years after chronic exposure. Radiographically it presents with multiple, small upper lung nodules. Pathologically silicosis reveals birefringent material surrounded by fibrosis. Acute silicosis (rare) develops weeks to years after massive exposures. Patients with acute silicosis present with bibasilar consolidations and severe shortness of breath. Silicosis confers a significant increase risk of mycobacterial infection, specifically tuberculosis. Any time a patient with silicosis presents with a new cavitary nodule or symptoms (fevers, weight loss, night sweats) active tuberculosis should be considered as these patients are at a significantly increased risk. Silicosis patients found to have active tuberculosis are treated for extended duration. PPD (a skin test for latent tuberculosis) testing is recommended in all patients with silicosis. Squamous cell carcinoma can cavitate but squamous cell carcinoma typically occurs in smokers. The combination of silica exposure plus tobacco exposure appears to be synergist. In this non-smoker with symptoms consistent with an infection, squamous cell cancer is unlikely. Progressive massive fibrosis can occur in silicosis or coal workers pneumoconiosis. The small nodules in the upper lobes of either disease coalesce into large nodule or mass that pull toward the hilum. The conglomerate of the nodules is often concerning for malignancy but progressive massive fibrosis is slow growing and not highly metabolic on Positron Emission Tomography (PET) scan. PET scans involve giving a patient radiolabeled sugar and observing which cells are highly metabolically active (taking up lots of sugar). PET scanning is used frequently during the diagnosis and staging of malignancy.

A 68-yr-old non-smoking sand blaster has developed chronic silicosis. His CT shows calcified hilar lymphadenopathy, multiple upper lobe predominant nodules that are stable over the last 12 months and a new 2.5 cm cavitary nodule in the right upper lobe along with fevers and chills. He has a cough productive of sputum but sputum gram stain is negative. What is the most likely cause of this cavitation? a. Squamous cell carcinoma b. Tuberculosis c. Progressive massive fibrosis d. Simple silicosis

d. Benign asbestos pleural effusion

A 75-year-old retired heating and air-conditioning service man presents for routine follow-up. He has known calcified pleural plaques due to exposure to asbestos. He reports no symptoms of chest pain or shortness of breath. He is a lifelong non-smoker. On exam he is found to have dullness to percussion of the right lung base and decreased breath sounds. Chest x-ray and CT confirm stable bilateral, calcified pleural plaques without any increase thickness or nodularity. There is a new small right sided pleural effusion. Thoracentesis is performed. Serum LDH 120 U/L (Normal 98-192) and Serum Protein 6.2 g/dL (Normal 5.9-7.5). Pleural fluid LDH 101 U/L and Pleural fluid Protein 3.6 g/dL. Cell count on the pleural fluid reveals an elevated WBC with an increased proportion of eosinophils at 40%. Pleural fluid cytology is negative. Pleural fluid gram stain and cultures are negative. What is the most likely diagnosis? a. Mesothelioma b. Parapneumonic effusion c. Stage IV lung adenocarcinoma d. Benign asbestos pleural effusion

d. Pneumothorax, no hypoechoic area, no pleural sliding, abnormal M pattern

A patient presents with decreased breath sounds of the right lung. Which is the correct grouping of ultrasound findings? a. Effusion, No hypoechoic area, pleural sliding, Normal M pattern b. Pneumothorax, hypoechoic area, no pleural sliding, abnormal M pattern c. consolidation, no hypoechoic area, no pleural sliding, normal m pattern d. Pneumothorax, no hypoechoic area, no pleural sliding, abnormal M pattern

c. Appendectomy 1 weeks ago and swollen left leg concerning for DVT The Well's criteria total is 4.5 = 1.5 (for surgery) and 3 (for clinical signs of DVT). Answer A score total of 3 = 1.5 and 1.5. Answer B score total of 2 = 1 and 1. Answer D score total of 3 = 1.5 and 1.5.

According to Well's scoring which of these patients is most likely to have a pulmonary embolism? a. Tachycardia to 120 and previous DVT b. Metastatic lung cancer and active hemoptysis c. Appendectomy 1 weeks ago and swollen left leg concerning for DVT d. Previous PE and immobilization for 5 days

d. Phrenic nerve

An 80-year-old female present with right sided chest pain. She is found to have a multiloculated pleural effusion on ultrasound (shown below). She has associated fevers, chills and productive cough. She says the pain is severe and radiates to her right shoulder. Pleural fluid analysis reveals a pleural fluid protein of 5 g/dL, LDH 275 U/L, pH 7.03, glucose 12 mg/dL and gram stain is positive for gram positive cocci in pairs. Which nerve is responsible for this patient's pain in her shoulder? a. Intercostal nerve b. Vagal nerve c. Long thoracic nerve d. Phrenic nerve

c. Dual circulation from pulmonary and bronchial circulation Answer C is correct. Pulmonary infarctions occur when there is no blood flow to an area of lung. Normally pulmonary oxygenation is achieved by both the bronchial artery circulation and the pulmonary artery circulation. This dual blood supply prevents pulmonary infarctions from occurring when one blood supply is interrupted as in pulmonary embolism. Patients with atherosclerotic vascular disease are at increased risk of pulmonary infarction due to the inability to increase flow through narrowed vessels. Hypoxia in the lung generates vasoconstriction, not vasodilation. This property is unique to the lung and helps achieve V/Q matching. Regional diffusion of oxygen cannot provide enough oxygen to prevent hypoxic injury. Leftward shift of the hemoglobin dissociation curve causes oxygen to be held onto with a higher affinity. Leftward shift of the hemoglobin dissociation curve occurs in alkalosis, hypothermia and low 2,3- Bisphosphoglyceric acid (DPG). Low levels of DPG are found in fetal Hgb thus fetal Hgb has high affinity for oxygen. A rightward shift of the hemoglobin dissociation curve is expected in ischemic tissue due to the acidosis that is generated. This results is less affinity for oxygen for easier offloading into the tissue.

Why are pulmonary infarctions uncommon in the setting of pulmonary embolism? a. Hypoxic vasodilation in the lung b. Adequate regional oxygenation from adjacent alveoli c. Dual circulation from pulmonary and bronchial circulation d. Leftward shift of the hemoglobin dissociation curve


Related study sets

Study Block 3: Chapter 20,21,19,18

View Set

Anatomy lect 15+, A&P Midterm 1, Anatomy Midterm 2

View Set

Complications Occurring Before Labor and Delivery

View Set